article 7 compiled digested cases

55
RUBIO, RAMIL C CONSTITUTIONAL LAW LLB-1C Marcos vs. Manglapus, G.R. no. 88211, September 15, 1989 Facts : In February 1986, Ferdinand E. Marcos was deposed from presidency via people power and forced into exile in Hawaii. Nearly three years after, in his deathbed seeks return to the Philippines to die. Thus, this petition for mandamus and prohibition asks the Court to order the respondent to issue travel documents to Mr. Marcos and the immediate members of his family and to enjoin the the implementation of President Aquino’s decision to bar their return to the Philippines. Petitioners contend that the right of the Marcoses to return to the Philippines is guaranteed under the Bill of Rights. That the President is without power to impair the liberty of abode of the Marcoses because only the court may do so “within the limits prescribed by law.” The President has enumerated powers and what is not enumerated is impliedly denied to her. Issues : Whether or not the President has the power under the Constitution, to bar Marcoses from returning to the Philippines Whether or not the President acted arbitrarily or with grave abuse of discretion amounting to lack or excess of jurisdiction upon determining that the Marcoses’s return poses a serious threat to national interest and welfare and decided to bar their return Held : The executive power of the President under the Constitution is more than the sum of specific powers enumerated under the Constitution. In balancing the general welfare and the common good against the exercise of rights of certain individuals, the power involved is the President’s residual power to protect the general welfare of the people. Presidential power is a wide discretion, within the bounds of laws and extraordinary in times of emergency. The President did not act arbitrarily or with grave abuse of discretion in determining that the return of former President Marcos and his family poses a serious threat to national interest and welfare. There exist factual bases in the President’s decision in the pleadings, oral arguments and facts filed by the parties during the briefing in chambers by the Chief of Staff of the Armed of the Philppines and National Security Adviser. That the President has the power under the Constitution to bar the Macrose’s from returning has been recognized by the members of the Legislature. Through a Resolution proposed in the House of Representative, signed by 103 members urging the President to allow Mr. Marcos to return to the Philippines –an act of true national reconciliation. The Resolution does not question the President’s power but was an appeal to allow a man to come home and to die in his country. Such request submit to the exercise of a broader discretion on the part of the President to determine whether it must be granted or not. The case is not a political question and for such, the court exercised its judicial power involving the determination whether there has been a grave abuse of discretion on the part of any branch or instrumnetality of the government. Laurel V Garcia Facts: Petitioners seek to stop the Philippine Government to sell the Roppongi Property, which is located in Japan. It is one of the properties given by the Japanese Government as reparations for damage done by the latter to the former during the war. Petitioner argues that under Philippine Law, the subject property is property of public dominion. As such, it is outside the commerce of men. Therefore, it cannot be alienated. Respondents aver that Japanese Law, and not Philippine Law, shall apply to the case because the property is located in Japan. They posit that the principle of lex situs applies.

Upload: princess-samourai

Post on 26-Dec-2015

35 views

Category:

Documents


0 download

DESCRIPTION

cases

TRANSCRIPT

Page 1: Article 7 Compiled Digested Cases

RUBIO, RAMIL C

CONSTITUTIONAL LAW

LLB-1C

Marcos vs. Manglapus, G.R. no. 88211, September 15, 1989

Facts : In February 1986, Ferdinand E. Marcos was deposed from presidency via people power and forced into exile in Hawaii. Nearly three years after, in his deathbed seeks return to the Philippines to die.

Thus, this petition for mandamus and prohibition asks the Court to order the respondent to issue travel documents to Mr. Marcos and the immediate members of his family and to enjoin the the implementation of President Aquino’s decision to bar their return to the Philippines.

Petitioners contend that the right of the Marcoses to return to the Philippines is guaranteed under the Bill of Rights. That the President is without power to impair the liberty of abode of the Marcoses because only the court may do so “within the limits prescribed by law.” The President has enumerated powers and what is not enumerated is impliedly denied to her.

Issues : Whether or not the President has the power under the Constitution, to bar Marcoses from returning to the Philippines

Whether or not the President acted arbitrarily or with grave abuse of discretion amounting to lack or excess of jurisdiction upon determining that the Marcoses’s return poses a serious threat to national interest and welfare and decided to bar their return

Held : The executive power of the President under the Constitution is more than the sum of specific powers enumerated under the Constitution. In balancing the general welfare and the common good against the exercise of rights of certain individuals, the power involved is the President’s residual power to protect the general welfare of the people. Presidential power is a wide discretion, within the bounds of laws and extraordinary in times of emergency.

The President did not act arbitrarily or with grave abuse of discretion in determining that the return of former President Marcos and his family poses a serious threat to national interest and welfare. There exist factual bases in the President’s decision in the pleadings, oral arguments and facts filed by the parties during the briefing in chambers by the Chief of Staff of the Armed of the Philppines and National Security Adviser.

That the President has the power under the Constitution to bar the Macrose’s from returning has been recognized by the members of the Legislature. Through a Resolution proposed in the House of Representative, signed by 103 members urging the President to allow Mr. Marcos to return to the Philippines –an act of true national reconciliation. The Resolution does not question the President’s power but was an appeal to allow a man to come home and to die in his country. Such request submit to the exercise of a broader discretion on the part of the President to determine whether it must be granted or not.

The case is not a political question and for such, the court exercised its judicial power involving the determination whether there has been a grave abuse of discretion on the part of any branch or instrumnetality of the government.

Laurel V Garcia

Facts:

Petitioners seek to stop the Philippine Government to sell the Roppongi Property, which is located in Japan. It is one of the properties given by the Japanese Government as reparations for damage done by the latter to the former during the war.

Petitioner argues that under Philippine Law, the subject property is property of public dominion. As such, it is outside the commerce of men. Therefore, it cannot be alienated.

Respondents aver that Japanese Law, and not Philippine Law, shall apply to the case because the property is located in Japan. They posit that the principle of lex situs applies.

Issues and Held:

1. WON the subject property cannot be alienated.

The answer is in the affirmative.

Under Philippine Law, there can be no doubt that it is of public dominion unless it is convincingly shown that the property has become patrimonial. This, the respondents have failed to do. As property of public dominion, the Roppongi lot is outside the commerce of man. It cannot be alienated.

2. WON Philippine Law applies to the case at bar.

The answer is in the affirmative.

We see no reason why a conflict of law rule should apply when no conflict of law situation exists. A conflict of law situation arises only when: (1) There is a dispute over the title or ownership of an immovable, such that the capacity to take and transfer immovables, the formalities of conveyance, the essential validity and effect of the transfer, or the interpretation and effect of a conveyance, are to be determined; and (2) A foreign law on land ownership and its conveyance is asserted to conflict with a domestic law on the same matters. Hence, the need to determine which law should apply.

Page 2: Article 7 Compiled Digested Cases

In the instant case, none of the above elements exists.

The issues are not concerned with validity of ownership or title. There is no question that the property belongs to the Philippines. The issue is the authority of the respondent officials to validly dispose of property belonging to the State. And the validity of the procedures adopted to effect its sale. This is governed by Philippine Law. The rule of lex situs does not apply.

The assertion that the opinion of the Secretary of Justice sheds light on the relevance of the lex situs rule is misplaced. The opinion does not tackle the alienability of the real properties procured through reparations nor the existence in what body of the authority to sell them. In discussing who are capable of acquiring the lots, the Secretary merely explains that it is the foreign law which should determine who can acquire the properties so that the constitutional limitation on acquisition of lands of the public domain to Filipino citizens and entities wholly owned by Filipinos is inapplicable.

Estrada V Disierto

[Immunity from Suit; Resignation of the President; Justiciable controversy]

FACTS:

It began in October 2000 when allegations of wrong doings involving bribe-taking, illegal gambling, and other forms of corruption were made against Estrada before the Senate Blue Ribbon Committee. On November 13, 2000, Estrada was impeached by the Hor and, on December 7, impeachment proceedings were begun in the Senate during which more serious allegations of graft and corruption against Estrada were made and were only stopped on January 16, 2001 when 11 senators, sympathetic to the President, succeeded in suppressing damaging evidence against Estrada. As a result, the impeachment trial was thrown into an uproar as the entire prosecution panel walked out and Senate President Pimentel resigned after casting his vote against Estrada.

On January 19, PNP and the AFP also withdrew their support for Estrada and joined the crowd at EDSA Shrine. Estrada called for a snap presidential election to be held concurrently with congressional and local elections on May 14, 2001. He added that he will not run in this election. On January 20, SC declared that the seat of presidency was vacant, saying that Estrada “constructively resigned his post”. At noon, Arroyo took her oath of office in the presence of the crowd at EDSA as the 14th President. Estrada and his family later left Malacañang Palace. Erap, after his fall, filed petition for prohibition with prayer for WPI. It sought to enjoin the respondent Ombudsman from “conducting any further proceedings in cases filed against him not until his term as president ends. He also prayed for judgment “confirming Estrada to be the lawful and incumbent President of the Republic of the Philippines temporarily unable to discharge the duties of his office.

ISSUE(S):

1. WoN the petition presents a justiciable controversy.

2. WoN Estrada resigned as President.

3. WoN Arroyo is only an acting President.

4. WoN the President enjoys immunity from suit.

5. WoN the prosecution of Estrada should be enjoined due to prejudicial publicity.

RULING:

1. Political questions- "to those questions which, under the Constitution, are to be decided by the people in their sovereign capacity, or in regard to which full discretionary authority has been delegated to the legislative or executive branch of the government. It is concerned with issues dependent upon the wisdom, not legality of a particular measure."

Legal distinction between EDSA People Power I EDSA People Power II:

EDSA I

EDSA II

exercise of the people power of revolution which overthrew the whole government.

exercise of people power of freedom of speech and freedom of assembly to petition the government for redress of grievances which only affected the office of the President.

extra constitutional and the legitimacy of the new government that resulted from it cannot be the subject of judicial review

intra constitutional and the resignation of the sitting President that it caused and the succession of the Vice President as President are subject to judicial review.

Page 3: Article 7 Compiled Digested Cases

presented a political question;

involves legal questions.

The cases at bar pose legal and not political questions. The principal issues for resolution require the proper interpretation of certain provisions in the 1987 Constitution: Sec 1 of Art II, and Sec 8 of Art VII, and the allocation of governmental powers under Sec 11 of Art VII. The issues likewise call for a ruling on the scope of presidential immunity from suit. They also involve the correct calibration of the right of petitioner against prejudicial publicity.

2. Elements of valid resignation: (a)an intent to resign and (b) acts of relinquishment. Both were present when President Estrada left the Palace.

Totality of prior contemporaneous posterior facts and circumstantial evidence— bearing material relevant issues—President Estrada is deemed to have resigned— constructive resignation.

SC declared that the resignation of President Estrada could not be doubted as confirmed by his leaving Malacañan Palace. In the press release containing his final statement:

1. He acknowledged the oath-taking of the respondent as President;

2. He emphasized he was leaving the Palace for the sake of peace and in order to begin the healing process (he did not say that he was leaving due to any kind of disability and that he was going to reassume the Presidency as soon as the disability disappears);

3. He expressed his gratitude to the people for the opportunity to serve them as President (without doubt referring to the past opportunity);

4. He assured that he will not shirk from any future challenge that may come in the same service of the country;

5. He called on his supporters to join him in promotion of a constructive national spirit of reconciliation and solidarity.

Intent to resign—must be accompanied by act of relinquishment—act or omission before, during and after January 20, 2001.

3. The Congress passed House Resolution No. 176 expressly stating its support to Gloria Macapagal-Arroyo as President of the Republic of the Philippines and subsequently passed H.R. 178 confirms the nomination of Teofisto T. Guingona Jr. As Vice President. Senate passed HR No. 83 declaring the Impeachment Courts as Functius Officio and has been terminated. It is clear is that both houses of Congress recognized Arroyo as the President. Implicitly clear in that recognition is the premise that the inability of Estrada is no longer temporary as the Congress has clearly rejected his claim of inability.

The Court therefore cannot exercise its judicial power for this is political in nature and addressed solely to Congress by constitutional fiat. In fine, even if Estrada can prove that he did not resign, still, he cannot successfully claim that he is a President on leave on the ground that he is merely unable to govern temporarily. That claim has been laid to rest by Congress and the decision that Arroyo is the de jure, president made by a co-equal branch of government cannot be reviewed by this Court.

4. The cases filed against Estrada are criminal in character. They involve plunder, bribery and graft and corruption. By no stretch of the imagination can these crimes, especially plunder which carries the death penalty, be covered by the alleged mantle of immunity of a non-sitting president. He cannot cite any decision of this Court licensing the President to commit criminal acts and wrapping him with post-tenure immunity from liability. The rule is that unlawful acts of public officials are not acts of the State and the officer who acts illegally is not acting as such but stands in the same footing as any trespasser.

5. No. Case law will tell us that a right to a fair trial and the free press are incompatible. Also, since our justice system does not use the jury system, the judge, who is a learned and legally enlightened individual, cannot be easily manipulated by mere publicity. The Court also said that Estrada did not present enough evidence to show that the publicity given the trial has influenced the judge so as to render the judge unable to perform. Finally, the Court said that the cases against Estrada were still undergoing preliminary investigation, so the publicity of the case would really have no permanent effect on the judge and that the prosecutor should be more concerned with justice and less with prosecution.

Balao V GMA

BALAO et al vs. GMA

G.R. No. 186050

December 13, 2011

FACTS: The siblings of James Balao, and Longid (petitioners), filed with the RTC of La Trinidad, Benguet a Petition for the Issuance of a Writ of Amparo in favor of James Balao who was abducted by unidentified armed men earlier. Named respondents in the petition were then President GMA, Exec Sec Eduardo Ermita, Defense Sec Gilberto Teodoro, Jr., ILG Secretary Ronaldo Puno, National Security Adviser (NSA) Norberto Gonzales, AFP Chief of Staff Gen. Alexander . Yano, PNP Police Director General Jesus Verzosa, among others.

Page 4: Article 7 Compiled Digested Cases

James M. Balao is a Psychology and Economics graduate of the UP-Baguio. In 1984, he was among those who founded the Cordillera Peoples Alliance (CPA), a coalition of NGOs working for the cause of indigenous peoples in the Cordillera Region.

According to witnesses’ testimony, James was abducted by unidentified men, saying they were policemen and were arresting him for a drugs case and then made to ride a white van.

petitioners prayed for the issuance of a writ of amparo and likewise prayed for (1) an inspection order for the inspection of at least 11 military and police facilities which have been previously reported as detention centers for activists abducted by military and police operatives; (2) a production order for all documents that contain evidence relevant to the petition, particularly the Order of Battle List and any record or dossier respondents have on James; and (3) a witness protection order.

the RTC issued the assailed judgment, disposing as follows:

ISSUE a Writ of Amparo Ordering the respondents to (a) disclose where James is detained or confined, (b) to release James considering his unlawful detention since his abduction and (c) to cease and desist from further inflicting harm upon his person; and

DENY the issuance of INSPECTION ORDER, PRODUCTION ORDER and WITNESS PROTECTION ORDER for failure of herein Petitioners to comply with the stringent provisions on the Rule on the Writ of Amparo and substantiate the same

ISSUE: WON the totality of evidence satisfies the degree of proof required by the Amparo Rule to establish an enforced disappearance.

HELD: NO; The Rule on the Writ of Amparo was promulgated on October 24, 2007 amidst rising incidence of “extralegal killings” and “enforced disappearances.” It was formulated in the exercise of this Court’s expanded rule-making power for the protection and enforcement of constitutional rights enshrined in the 1987 Constitution, albeit limited to these two situations. “Extralegal killings” refer to killings committed without due process of law, i.e., without legal safeguards or judicial proceedings. On the other hand, “enforced disappearances” are attended by the following characteristics: an arrest, detention, or abduction of a person by a government official or organized groups or private individuals acting with the direct or indirect acquiescence of the government; the refusal of the State to disclose the fate or whereabouts of the person concerned or a refusal to acknowledge the deprivation of liberty which places such person outside the protection of law.

**

The trial court gave considerable weight to the discussion in the petition of briefing papers supposedly obtained from the AFP indicating that the anti-insurgency campaign of the military under the administration of President Arroyo included targeting of identified legal organizations under the NDF, which included the CPA, and their members, as “enemies of the state.

We hold that such documented practice of targeting activists in the military’s counter-insurgency program by itself does not fulfill the evidentiary standard provided in the Amparo Rule to establish an enforced disappearance.

In the case of Roxas v. Macapagal-Arroyo, the Court noted that the similarity between the circumstances attending a particular case of abduction with those surrounding previous instances of enforced disappearances does not, necessarily, carry sufficient weight to prove that the government orchestrated such abduction. Accordingly, the trial court in this case cannot simply infer government involvement in the abduction of James from past similar incidents in which the victims also worked or affiliated with the CPA and other left-leaning groups.

**

The petition further premised government complicity in the abduction of James on the very positions held by the respondents. The Court in Rubrico v. Macapagal-Arroyo had the occasion to expound on the doctrine of command responsibility and why it has little bearing, if at all, in amparo proceedings.

It may plausibly be contended that command responsibility, as legal basis to hold military/police commanders liable for extra-legal killings, enforced disappearances, or threats, may be made applicable to this jurisdiction on the theory that the command responsibility doctrine now constitutes a principle of international law or customary international law in accordance with the incorporation clause of the Constitution. Still, it would be inappropriate to apply to these proceedings the doctrine of command responsibility, as the CA seemed to have done, as a form of criminal complicity through omission, for individual respondents’ criminal liability, if there be any, is beyond the reach of amparo. In other words, the Court does not rule in such proceedings on any issue of criminal culpability, even if incidentally a crime or an infraction of an administrative rule may have been committed. As the Court stressed in Secretary of National Defense v. Manalo (Manalo), the writ of amparo was conceived to provide expeditious and effective procedural relief against violations or threats of violation of the basic rights to life, liberty, and security of

Page 5: Article 7 Compiled Digested Cases

persons; the corresponding amparo suit, however, “is not an action to determine criminal guilt requiring proof beyond reasonable doubt x x x or administrative liability requiring substantial evidence that will require full and exhaustive proceedings.” Of the same tenor, and by way of expounding on the nature and role of amparo, is what the Court said in Razon v. Tagitis:

It does not determine guilt nor pinpoint criminal culpability for the disappearance [threats thereof or extrajudicial killings]; it determines responsibility, or at least accountability, for the enforced disappearance [threats thereof or extrajudicial killings] for purposes of imposing the appropriate remedies to address the disappearance [or extrajudicial killings].

x x x x

As the law now stands, extrajudicial killings and enforced disappearances in this jurisdiction are not crimes penalized separately from the component criminal acts undertaken to carry out these killings and enforced disappearances and are now penalized under the Revised Penal Code and special laws. The simple reason is that the Legislature has not spoken on the matter; the determination of what acts are criminal x x x are matters of substantive law that only the Legislature has the power to enact. x x x[

Assessing the evidence on record, we find that the participation in any manner of military and police authorities in the abduction of James has not been adequately proven. The identities of the abductors have not been established, much less their link to any military or police unit. There is likewise no concrete evidence indicating that James is being held or detained upon orders of or with acquiescence of government agents. Consequently, the trial court erred in granting amparo reliefs. Such pronouncement of responsibility on the part of public respondents cannot be made given the insufficiency of evidence. However, we agree with the trial court in finding that the actions taken by respondent officials are “very limited, superficial and one-sided.” Its candid and forthright observations on the efforts exerted by the respondents are borne by the evidence on record.

**

An inspection order is an interim relief designed to give support or strengthen the claim of a petitioner in an amparo petition, in order to aid the court before making a decision. A basic requirement before an amparo court may grant an inspection order is that the place to be inspected is reasonably determinable from the allegations of the party seeking the order. In this case, the issuance of inspection order was properly denied since the petitioners specified several military and police establishments based merely on the allegation that the testimonies of victims and witnesses in previous incidents of similar abductions involving activists disclosed that those premises were used as detention centers. In the same vein, the prayer for issuance of a production order was predicated on petitioners’ bare allegation that it obtained confidential information from an unidentified military source, that the name of James was included in the so-called Order of Battle. Indeed, the trial court could not have sanctioned any “fishing expedition” by precipitate issuance of inspection and production orders on the basis of insufficient claims of one party.

Maximo V Soliven

167 SCRA 393 – Political Law – Constitutional Law – President’s Immunity From Suit – Must Be Invoked by the President

Luis Beltran is among the petitioners in this case. He, together with others, was charged with libel by the then president Corzaon

Aquino. Cory herself filed a complaint-affidavit against him and others. Makasiar averred that Cory cannot file a complaint affidavit

because this would defeat her immunity from suit. He grounded his contention on the principle that a president cannot be sued.

However, if a president would sue then the president would allow herself to be placed under the court’s jurisdiction and conversely she

would be consenting to be sued back. Also, considering the functions of a president, the president may not be able to appear in court to

be a witness for herself thus she may be liable for contempt.

ISSUE: Whether or not such immunity can be invoked by Beltran, a person other than the president.

HELD: No. The rationale for the grant to the President of the privilege of immunity from suit is to assure the exercise of Presidential

duties and functions free from any hindrance or distraction, considering that being the Chief Executive of the Government is a job that,

aside from requiring all of the office-holder’s time, also demands undivided attention.

But this privilege of immunity from suit, pertains to the President by virtue of the office and may be invoked only by the holder of the

office; not by any other person in the President’s behalf. Thus, an accused like Beltran et al, in a criminal case in which the President is

the complainant cannot raise the presidential privilege as a defense to prevent the case from proceeding against such accused.

Moreover, there is nothing in our laws that would prevent the President from waiving the privilege. Thus, if so minded the President may

shed the protection afforded by the privilege and submit to the court’s jurisdiction. The choice of whether to exercise the privilege or to

waive it is solely the President’s prerogative. It is a decision that cannot be assumed and imposed by any other person.

Senate vs. Ermita , GR 169777, April 20, 2006

Senate vs. Ermita , GR 169777, April 20, 2006

Page 6: Article 7 Compiled Digested Cases

FACTS:

This is a petition for certiorari and prohibition proffer that the President has abused power by issuing E.O. 464 “Ensuring Observance of the Principles of Separation of Powers, Adherence to the Rule on Executive Privilege and Respect for the Rights of Public Officials Appearing in Legislative Inquiries in Aid of Legislation Under the Constitution, and for Other Purposes”. Petitioners pray for its declaration as null and void for being unconstitutional.

In the exercise of its legislative power, the Senate of the Philippines, through its various Senate Committees, conducts inquiries or investigations in aid of legislation which call for, inter alia, the attendance of officials and employees of the executive department, bureaus, and offices including those employed in Government Owned and Controlled Corporations, the Armed Forces of the Philippines (AFP), and the Philippine National Police (PNP).

The Committee of the Senate issued invitations to various officials of the Executive Department for them to appear as resource speakers in a public hearing on the railway project, others on the issues of massive election fraud in the Philippine elections, wire tapping, and the role of military in the so-called “Gloriagate Scandal”.

Said officials were not able to attend due to lack of consent from the President as provided by E.O. 464, Section 3 which requires all the public officials enumerated in Section 2(b) to secure the consent of the President prior to appearing before either house of Congress.

ISSUE:

Is Section 3 of E.O. 464, which requires all the public officials, enumerated in Section 2(b) to secure the consent of the President prior to appearing before either house of Congress, valid and constitutional?

RULING:

No. The enumeration in Section 2 (b) of E.O. 464 is broad and is covered by the executive privilege. The doctrine of executive privilege is premised on the fact that certain information must, as a matter of necessity, be kept confidential in pursuit of the public interest. The privilege being, by definition, an exemption from the obligation to disclose information, in this case to Congress, the necessity must be of such high degree as to outweigh the public interest in enforcing that obligation in a particular case.

Congress undoubtedly has a right to information from the executive branch whenever it is sought in aid of legislation. If the executive branch withholds such information on the ground that it is privileged, it must so assert it and state the reason therefor and why it must be respected.

The infirm provisions of E.O. 464, however, allow the executive branch to evade congressional requests for information without need of clearly asserting a right to do so and/or proffering its reasons therefor. By the mere expedient of invoking said provisions, the power of Congress to conduct inquiries in aid of legislation is frustrated.

CASE DIGEST - AKBAYAN VS. AQUINO

Facts: The signing of the Japan-Philippines Economic Partnership Agreement (JPEPA) at the sidelines of the Asia-Europe Summit in Helsinki in September 2006 was hailed by both Japanese Prime Minister Junichiro Koizumi and Philippine President Gloria Macapagal Arroyo as a “milestone in the continuing cooperation and collaboration, setting a new chapter of strategic partnership for mutual opportunity and growth (for both countries).”

JPEPA which has been referred to as a ‘mega treaty’ is a comprehensive plan for opening up of markets in goods and services as well as removing barriers and restrictions on investments. It is a deal that encompasses even our commitments to the WTO.

The complexity of JPEPA became all the more evident at the Senate hearing conducted by the Committee on Trade and Commerce last November 2006. The committee, chaired by Senator Mar Roxas, heard differing views and perspectives on JPEPA. On one hand the committee heard Government’s rosy projections on the economic benefits of JPEPA and on the other hand the views of environmental and trade activists who raised there very serious concerns about the country being turned into Japan’s toxic waste basket. The discussion in the Senate showed that JPEPA is not just an issue concerning trade and economic relations with Japan but one that touches on broader national development concerns.

Issues:

1. Do the therein petitioners have standing to bring this action for mandamus in their capacity as citizens of the Republic, as taxpayers, and as members of the Congress

Page 7: Article 7 Compiled Digested Cases

2. Can this Honorable Court exercise primary jurisdiction of this case and take cognizance of the instant petition.

3. Are the documents and information being requested in relation to the JPEPA exempted from the general rules on transparency and full public disclosure such that the Philippine government is justified in denying access thereto.

Rulings:

The Supreme Court en banc promulgated last July 16, 2008 its ruling on the case of “Akbayan Citizens Action Party et al vs. Thomas G. Aquino et al” (G.R. No. 170516). The Highest Tribunal dismissed the Petition for mandamus and prohibition, which sought to compel respondents Department of Trade Industry (DTI) Undersecretary Thomas Aquino et al to furnish petitioners the full text of the Japan-Philippines Economic Partnership Agreement (JPEPA) and the lists of the Philippine and Japanese offers submitted during the negotiation process and all pertinent attachments and annexes thereto.

In its Decision, the Court noted that the full text of the JPEPA has been made accessible to the public since 11 September 2006, and thus the demand to be furnished with copy of the said document has become moot and academic. Notwithstanding this, however, the Court lengthily discussed the substatives issues, insofar as they impinge on petitioners' demand for access to the Philippine and Japanese offers in the course of the negotiations.

The Court held: “Applying the principles adopted in PMPF v. Manglapus, it is clear that while the final text of the JPEPA may not be kept perpetually confidential – since there should be 'ample opportunity for discussion before [a treaty] is approved' – the offers exchanged by the parties during the negotiations continue to be privileged even after the JPEPA is published. It is reasonable to conclude that the Japenese representatives submitted their offers with the understanding that 'historic confidentiality' would govern the same. Disclosing these offers could impair the ability of the Philippines to deal not only with Japan but with other foreign governments in future negotiations.”

It also reasoned out that opening for public scrutiny the Philippine offers in treaty negotiations would discourage future Philippine representatives from frankly expressing their views during negotiations. The Highest Tribunal recognized that treaty negotiations normally involve a process of quid pro quo, where negotiators would willingly grant concessions in an area of lesser importance in order to obtain more favorable terms in an area of greater national interest.

In the same Decision, the Court took time to address the dissent of Chief Justice Reynato S. Puno. It said: “We are aware that behind the dissent of the Chief Justice lies a genuine zeal to protect our people's right to information against any abuse of executive privilege. It is a zeal that We fully share. The Court, however, in its endeavour to guard against the abuse of executive privilege, should be careful not to veer towards the opposite extreme, to the point that it would strike down as invalid even a legitimate exercise thereof.”

NERI VS. SENATE COMMITTEE

ROMULO L. NERI, petitioner vs. SENATE COMMITTEE ON ACCOUNTABILITY OF PUBLIC OFFICERS AND INVESTIGATIONS, SENATE COMMITTEE ON TRADE AND COMMERCE, AND SENATE COMMITTEE ON NATIONAL DEFENSE AND SECURITY

G.R. No. 180643, March 25, 2008

FACTS: On April 21, 2007, the Department of Transportation and Communication (DOTC) entered into a contract with Zhong Xing Telecommunications Equipment (ZTE) for the supply of equipment and services for the National Broadband Network (NBN) Project in the amount of U.S. $ 329,481,290 (approximately P16 Billion Pesos). The Project was to be financed by the People’s Republic of China.

The Senate passed various resolutions relative to the NBN deal. In the September 18, 2007 hearing Jose de Venecia III testified that several high executive officials and power brokers were using their influence to push the approval of the NBN Project by the NEDA.

Neri, the head of NEDA, was then invited to testify before the Senate Blue Ribbon. He appeared in one hearing wherein he was interrogated for 11 hrs and during which he admitted that Abalos of COMELEC tried to bribe him with P200M in exchange for his approval of the NBN project. He further narrated that he informed President Arroyo about the bribery attempt and that she instructed him not to accept the bribe.

Page 8: Article 7 Compiled Digested Cases

However, when probed further on what they discussed about the NBN Project, petitioner refused to answer, invoking “executive privilege”. In particular, he refused to answer the questions on:

(a) whether or not President Arroyo followed up the NBN Project,

(b) whether or not she directed him to prioritize it, and

(c) whether or not she directed him to approve.

He later refused to attend the other hearings and Ermita sent a letter to the senate averring that the communications between GMA and Neri are privileged and that the jurisprudence laid down in Senate vs Ermita be applied. He was cited in contempt of respondent committees and an order for his arrest and detention until such time that he would appear and give his testimony.

ISSUE:

Are the communications elicited by the subject three (3) questions covered by executive privilege?

HELD:

The communications are covered by executive privilege

The revocation of EO 464 (advised executive officials and employees to follow and abide by the Constitution, existing laws and jurisprudence, including, among others, the case of Senate v. Ermita when they are invited to legislative inquiries in aid of legislation.), does not in any way diminish the concept of executive privilege. This is because this concept has Constitutional underpinnings.

The claim of executive privilege is highly recognized in cases where the subject of inquiry relates to a power textually committed by the Constitution to the President, such as the area of military and foreign relations. Under our Constitution, the President is the repository of the commander-in-chief, appointing, pardoning, and diplomatic powers. Consistent with the doctrine of separation of powers, the information relating to these powers may enjoy greater confidentiality than others.

Several jurisprudence cited provide the elements of presidential communications privilege:

1) The protected communication must relate to a “quintessential and non-delegable presidential power.”

2) The communication must be authored or “solicited and received” by a close advisor of the President or the President himself. The judicial test is that an advisor must be in “operational proximity” with the President.

3) The presidential communications privilege remains a qualified privilege that may be overcome by a showing of adequate need, such that the information sought “likely contains important evidence” and by the unavailability of the information elsewhere by an appropriate investigating authority.

In the case at bar, Executive Secretary Ermita premised his claim of executive privilege on the ground that the communications elicited by the three (3) questions “fall under conversation and correspondence between the President and public officials” necessary in “her executive and policy decision-making process” and, that “the information sought to be disclosed might impair our diplomatic as well as economic relations with the People’s Republic of China.” Simply put, the bases are presidential communications privilege and executive privilege on matters relating to diplomacy or foreign relations.

Using the above elements, we are convinced that, indeed, the communications elicited by the three (3) questions are covered by the presidential communications privilege. First, the communications relate to a “quintessential and non-delegable power” of the President, i.e. the power to enter into an executive agreement with other countries. This authority of the President to enter into executive agreements without the concurrence of the Legislature has traditionally been recognized in Philippine jurisprudence. Second, the communications are “received” by a close advisor of the President. Under the “operational proximity” test, petitioner can be considered a close advisor, being a member of President Arroyo’s cabinet. And third, there is no adequate showing of a compelling need that would justify the limitation of the privilege and of the unavailability of the information elsewhere by an appropriate investigating authority.

Page 9: Article 7 Compiled Digested Cases

Respondent Committees further contend that the grant of petitioner’s claim of executive privilege violates the constitutional provisions on the right of the people to information on matters of public concern.50 We might have agreed with such contention if petitioner did not appear before them at all. But petitioner made himself available to them during the September 26 hearing, where he was questioned for eleven (11) hours. Not only that, he expressly manifested his willingness to answer more questions from the Senators, with the exception only of those covered by his claim of executive privilege.

The right to public information, like any other right, is subject to limitation. Section 7 of Article III provides:

The right of the people to information on matters of public concern shall be recognized. Access to official records, and to documents, and papers pertaining to official acts, transactions, or decisions, as well as to government research data used as basis for policy development, shall be afforded the citizen, subject to such limitations as may be provided by law.

Province of North Cotabato vs GRP Peace Panel on Ancestral Domain

Chester Cabalza recommends his visitors to please read the original & full text of the case cited. Xie xie!

Province of North Cotabato vs GRP Peace Panel on Ancestral Domain

G.R. No. 1833591,

October 14, 2008

Decision:

CARPIO MORALES, J.:

Subject of these consolidated cases is the extent of the powers of the President in pursuing the peace process. While the facts surrounding this controversy center on the armed conflict in Mindanao between the government and the Moro Islamic Liberation Front (MILF), the legal issue involved has a bearing on all areas in the country where there has been a long-standing armed conflict. Yet again, the Court is tasked to perform a delicate balancing act. It must uncompromisingly delineate the bounds within which the President may lawfully exercise her discretion, but it must do so in strict adherence to the Constitution, lest its ruling unduly restricts the freedom of action vested by that same Constitution in the Chief Executive precisely to enable her to pursue the peace process effectively.

Facts:

On August 5, 2008, the Government of the Republic of the Philippines (GRP) and the MILF, through the Chairpersons of their respective peace negotiating panels, were scheduled to sign a Memorandum of Agreement on the Ancestral Domain (MOA-AD) Aspect of the GRP-MILF Tripoli Agreement on Peace of 2001 in Kuala Lumpur, Malaysia.

The signing of the MOA-AD between the GRP and the MILF was not to materialize, however, for upon motion of petitioners, specifically those who filed their cases before the scheduled signing of the MOA-AD, this Court issued a Temporary Restraining Order enjoining the GRP from signing the same.

The MOA-AD was preceded by a long process of negotiation and the concluding of several prior agreements between the two parties beginning in 1996, when the GRP-MILF peace negotiations began. On July 18, 1997, the GRP and MILF Peace Panels signed the Agreement on General Cessation of Hostilities. The following year, they signed the General Framework of Agreement of Intent on August 27, 1998.

On July 23, 2008, the Province of North Cotabato and Vice-Governor Emmanuel Piñol filed a petition, docketed as G.R. No. 183591, for Mandamus and Prohibition with Prayer for the Issuance of Writ of Preliminary Injunction and Temporary Restraining Order. Invoking the right to information on matters of public concern, petitioners seek to compel respondents to disclose and furnish them the complete and official copies of the MOA-AD

Page 10: Article 7 Compiled Digested Cases

including its attachments, and to prohibit the slated signing of the MOA-AD, pending the disclosure of the contents of the MOA-AD and the holding of a public consultation thereon. Supplementarily, petitioners pray that the MOA-AD be declared unconstitutional.

Issues:

1. Whether the petitions have become moot and academic

(i) insofar as the mandamus aspect is concerned, in view of the disclosure of official copies of the final draft of the Memorandum of Agreement (MOA); and

(ii) insofar as the prohibition aspect involving the Local Government Units is concerned, if it is considered that consultation has become fait accompli with the finalization of the draft;

2. Whether the constitutionality and the legality of the MOA is ripe for adjudication;

3. Whether respondent Government of the Republic of the Philippines Peace Panel committed grave abuse of discretion amounting to lack or excess of jurisdiction when it negotiated and initiated the MOA vis-à-vis ISSUES Nos. 4 and 5;

4. Whether there is a violation of the people's right to information on matters of public concern (1987 Constitution, Article III, Sec. 7) under a state policy of full disclosure of all its transactions involving public interest (1987 Constitution, Article II, Sec. 28) including public consultation under Republic Act No. 7160 (LOCAL GOVERNMENT CODE OF 1991)[;]

If it is in the affirmative, whether prohibition under Rule 65 of the 1997 Rules of Civil Procedure is an appropriate remedy;

5. Whether by signing the MOA, the Government of the Republic of the Philippines would be BINDING itself

a) to create and recognize the Bangsamoro Juridical Entity (BJE) as a separate state, or a juridical, territorial or political subdivision not recognized by law;

b) to revise or amend the Constitution and existing laws to conform to the MOA;

c) to concede to or recognize the claim of the Moro Islamic Liberation Front for ancestral domain in violation of Republic Act No. 8371 (THE INDIGENOUS PEOPLES RIGHTS ACT OF 1997), particularly Section 3(g) & Chapter VII (DELINEATION, RECOGNITION OF ANCESTRAL DOMAINS)[;]

If in the affirmative, whether the Executive Branch has the authority to so bind the Government of the Republic of the Philippines;

6. Whether the inclusion/exclusion of the Province of North Cotabato, Cities of Zamboanga, Iligan and Isabela, and the Municipality of Linamon, Lanao del Norte in/from the areas covered by the projected Bangsamoro Homeland is a justiciable question; and

7. Whether desistance from signing the MOA derogates any prior valid commitments of the Government of the Republic of the Philippines.

Held:

Page 11: Article 7 Compiled Digested Cases

The main body of the MOA-AD is divided into four strands, namely, Concepts and Principles, Territory, Resources, and Governance.

The power of judicial review is limited to actual cases or controversies. Courts decline to issue advisory opinions or to resolve hypothetical or feigned problems, or mere academic questions. The limitation of the power of judicial review to actual cases and controversies defines the role assigned to the judiciary in a tripartite allocation of power, to assure that the courts will not intrude into areas committed to the other branches of government.

As the petitions involve constitutional issues which are of paramount public interest or of transcendental importance, the Court grants the petitioners, petitioners-in-intervention and intervening respondents the requisite locus standi in keeping with the liberal stance adopted in David v. Macapagal-Arroyo.

Contrary to the assertion of respondents that the non-signing of the MOA-AD and the eventual dissolution of the GRP Peace Panel mooted the present petitions, the Court finds that the present petitions provide an exception to the "moot and academic" principle in view of (a) the grave violation of the Constitution involved; (b) the exceptional character of the situation and paramount public interest; (c) the need to formulate controlling principles to guide the bench, the bar, and the public; and (d) the fact that the case is capable of repetition yet evading review.

The MOA-AD is a significant part of a series of agreements necessary to carry out the GRP-MILF Tripoli Agreement on Peace signed by the government and the MILF back in June 2001. Hence, the present MOA-AD can be renegotiated or another one drawn up that could contain similar or significantly dissimilar provisions compared to the original.

That the subject of the information sought in the present cases is a matter of public concern faces no serious challenge. In fact, respondents admit that the MOA-AD is indeed of public concern. In previous cases, the Court found that the regularity of real estate transactions entered in the Register of Deeds, the need for adequate notice to the public of the various laws, the civil service eligibility of a public employee, the proper management of GSIS funds allegedly used to grant loans to public officials, the recovery of the Marcoses' alleged ill-gotten wealth, and the identity of party-list nominees, among others, are matters of public concern. Undoubtedly, the MOA-AD subject of the present cases is of public concern, involving as it does the sovereignty and territorial integrity of the State, which directly affects the lives of the public at large.

In sum, the Presidential Adviser on the Peace Process committed grave abuse of discretion when he failed to carry out the pertinent consultation process, as mandated by E.O. No. 3, Republic Act No. 7160, and Republic Act No. 8371. The furtive process by which the MOA-AD was designed and crafted runs contrary to and in excess of the legal authority, and amounts to a whimsical, capricious, oppressive, arbitrary and despotic exercise thereof. It illustrates a gross evasion of positive duty and a virtual refusal to perform the duty enjoined.

The MOA-AD cannot be reconciled with the present Constitution and laws. Not only its specific provisions but the very concept underlying them, namely, the associative relationship envisioned between the GRP and the BJE, are unconstitutional, for the concept presupposes that the associated entity is a state and implies that the same is on its way to independence.

The Memorandum of Agreement on the Ancestral Domain Aspect of the GRP-MILF Tripoli Agreement on Peace of 2001 is declared contrary to law and the Constitution.

Phil Consti Asso V Enriquez

235 SCRA 506 – Political Law – Veto Power – Part of the Legislative Process

Constitutionality of the Pork Barrel “Countrywide Development Fund”

This is a consolidation of cases which sought to question the veto authority of the president involving the General Appropriations Act of

1994 as well as the constitutionality of the pork barrel. The Philippine Constitution Association (PHILCONSA) questions the countrywide

development fund. PHILCONSA said that Congress can only allocate funds but they cannot specify the items as to which those funds

would be applied for since that is already the function of the executive.

In G.R. No. 113766, after the vetoing by the president of some provisions of the GAA of 1994, neither house of congress took steps to

override the veto. Instead, Senators Tañada and Romulo sought the issuance of the writs of prohibition and mandamus against the

respondents in G.R. No. 113766. In this petition, petitioners contest the constitutionality of: (1) the veto on four special provisions added

to items in the GAA of 1994 for the Armed Forces of the Philippines (AFP) and the Department of Public Works and Highways (DPWH);

Page 12: Article 7 Compiled Digested Cases

and (2) the conditions imposed by the President in the implementation of certain appropriations for the CAFGU’s, the DPWH, and the

National Housing Authority (NHA).

ISSUE: Whether or not the President’s veto is valid.

HELD: In the PHILCONSA petition, the SC ruled that Congress acted within its power and that the CDF is constitutional. In the Tañada

petitions the SC dismissed the other petitions and granted the others.

Veto on special provisions

The president did his veto with certain conditions and compliant to the ruling inGonzales vs Macaraig. The president particularly vetoed

the debt reduction scheme in the GAA of 1994 commenting that the scheme is already taken cared of by other legislation and may be

more properly addressed by revising the debt policy. He, however did not delete the P86,323,438,000.00 appropriation therefor.

Tañada et al averred that the president cannot validly veto that provision w/o vetoing the amount allotted therefor. The veto of the

president herein is sustained for the vetoed provision is considered “inappropriate”; in fact the Sc found that such provision if not vetoed

would in effect repeal the Foreign Borrowing Act making the legislation as a log-rolling legislation.

Veto of provisions for revolving funds of SUCs

The appropriation for State Universities and Colleges (SUC’s), the President vetoed special provisions which authorize the use of

income and the creation, operation and maintenance of revolving funds was likewise vetoed. The reason for the veto is that there were

already funds allotted for the same in the National expenditure Program. Tañada et al claimed this as unconstitutional. The SC ruled

that the veto is valid for it is in compliant to the “One Fund Policy” – it avoided double funding and redundancy.

Veto of provision on 70% (administrative)/30% (contract) ratio for road maintenance

The President vetoed this provision on the basis that it may result to a breach of contractual obligations. The funds if allotted may result

to abandonment of some existing contracts. The SC ruled that this Special Provision in question is not an inappropriate provision which

can be the subject of a veto. It is not alien to the appropriation for road maintenance, and on the other hand, it specifies how the said

item shall be expended – 70% by administrative and 30% by contract.  The 1987 Constitution allows the addition by Congress of

special provisions, conditions to items in an expenditure bill, which cannot be vetoed separately from the items to which they relate so

long as they are “appropriate” in the budgetary sense. The veto herein is then not valid.

Veto of provision on prior approval of Congress for purchase of military equipment

As reason for the veto, the President stated that the said condition and prohibition violate the Constitutional mandate of non-impairment

of contractual obligations, and if allowed, “shall effectively alter the original intent of the AFP Modernization Fund to cover all military

equipment deemed necessary to modernize the AFP”. The SC affirmed the veto. Any provision blocking an administrative action in

implementing a law or requiring legislative approval of executive acts must be incorporated in a separate and substantive bill.

Therefore, being “inappropriate” provisions.

Veto of provision on use of savings to augment AFP pension funds

According to the President, the grant of retirement and separation benefits should be covered by direct appropriations specifically

approved for the purpose pursuant to Section 29(1) of Article VI of the Constitution. Moreover, he stated that the authority to use

savings is lodged in the officials enumerated in Section 25(5) of Article VI of the Constitution. The SC retained the veto per reasons

provided by the president.

Condition on the deactivation of the CAFGU’s

Congress appropriated compensation for the CAFGU’s including the payment of separation benefits. The President declared in his Veto

Message that the implementation of this Special Provision to the item on the CAFGU’s shall be subject to prior Presidential approval

pursuant to P.D. No. 1597 and R.A. No. 6758. The SC ruled to retain the veto per reasons provided by the president. Further, if this

provision is allowed the it would only lead to the repeal of said existing laws.

Conditions on the appropriation for the Supreme Court, etc

In his veto message: “The said condition is consistent with the Constitutional injunction prescribed under Section 8, Article IX-B of the

Constitutional which states that ‘no elective or appointive public officer or employee shall receive additional, double, or indirect

compensation unless specifically authorized by law.’ I am, therefore, confident that the heads of the said offices shall maintain fidelity to

the law and faithfully adhere to the well-established principle on compensation standardization. Tañada et al claim that the conditions

imposed by the President violated the independence and fiscal autonomy of the Supreme court, the Ombudsman, the COA and the

CHR. The SC sustained the veto: In the first place, the conditions questioned by petitioners were placed in the GAB by Congress itself,

not by the President. The Veto Message merely highlighted the Constitutional mandate that additional or indirect compensation can

only be given pursuant to law. In the second place, such statements are mere reminders that the disbursements of appropriations must

be made in accordance with law. Such statements may, at worse, be treated as superfluities.

Pork Barrel Constitutional

The pork barrel makes the unequal equal. The Congressmen, being representatives of their local districts know more about the

problems in their constituents areas than the national government or the president for that matter. Hence, with that knowledge, the

Congressmen are in a better position to recommend as to where funds should be allocated.

G.R. No. 121234, August 23, 1995

HUBERT J. P. WEBB, petitioner

VS.

HONORABLE RAUL E. DE LEON, the Presiding Judge of the Regional Trial Court of Parañaque, Branch 258,HONORABLE ZOSIMO

V. ESCANO, the Presiding Judge of the Regional Trial Court of Parañaque, Branch 259, PEOPLE OF THE PHILIPPINES, ZENON L.

DE GUIYAB, JOVENCITO ZUÑO, LEONARDO GUIYAB, JR., ROBERTO LAO,PABLO FORMARAN, and NATIONAL BUREAU OF

Page 13: Article 7 Compiled Digested Cases

INVESTIGATION, and HONORABLE AMELITA G. TOLENTINO, the Presiding Judge of the Regional Trial Court of Parañaque,

Branch 274, respondents

LAURO VIZCONDE, intervenor

FACTS:

On June 19, 1994, the National Bureau of Investigation (NBI) filed with the Department of Justice a letter-complaint charging petitioners

Hubert Webb, Michael Gatchalian, Antonio J. Lejano and six (6) other persons with the crime of Rape and Homicide of Carmela N.

Vizconde, her mother Estrellita Nicolas-Vizconde, and her sister Anne Marie Jennifer in their home at Number 80 W. Vinzons, St., BF

Homes Paranaque, Metro Manila on June 30, 1991.

Forthwith, the Department of Justice formed a panel of prosecutors headed by Assistant Chief State Prosecutor Jovencio R. Zuno to

conduct the preliminary investigation.

ARGUMENTS:

Petitioners fault the DOJ Panel for its finding of probable cause. They assail the credibility of Jessica Alfaro as inherently weak and

uncorroborated due to the inconsistencies between her April 28, 1995 and May 22, 1995 sworn statements. They criticize the

procedure followed by the DOJ Panel when it did not examine witnesses to clarify the alleged inconsistencies.

Petitioners charge that respondent Judge Raul de Leon and, later, respondent Judge Amelita Tolentino issued warrants of arrest

against them without conducting the required preliminary examination.

Petitioners complain about the denial of their constitutional right to due process and violation of their right to an impartial investigation.

They also assail the prejudicial publicity that attended their preliminary investigation.

ISSUES:

1. Whether or not the DOJ Panel likewise gravely abused its discretion in holding that there is probable cause to charge them with

the crime of rape and homicide

2. Whether or not respondent Judges de Leon and Tolentino gravely abused their discretion when they failed to conduct a preliminary

examination before issuing warrants of arrest against them

3. Whether or not the DOJ Panel denied them their constitutional right to due process during their preliminary investigation

4. Whether or not the DOJ Panel unlawfully intruded into judicial prerogative when it failed to charge Jessica Alfaro in the information

as an accused.

HELD:

1. NO.

2. NO.

3. NO. There is no merit in this contention because petitioners were given all the opportunities to be heard.

4. NO.

REASONS:

1. The Court ruled that the DOJ Panel did not gravely abuse its discretion when it found probable cause against the petitioners. A

probable cause needs only to rest on evidence showing that more likely than not, a crime has been committed and was committed

by the suspects. Probable cause need not be based on clear and convincing evidence of guilt, neither on evidence establishing

guilt beyond reasonable doubt and definitely, not on evidence establishing absolute certainty of guilt.

2. The Court ruled that respondent judges did not gravely abuse their discretion. In arrest cases, there must be a probable cause that

a crime has been committed and that the person to be arrested committed it. Section 6 of Rule 112 simply provides that “upon

filing of an information, the Regional Trial Court may issue a warrant for the accused. Clearly the, our laws repudiate the

submission of petitioners that respondent judges should have conducted “searching examination of witnesses” before issuing

warrants of arrest against them.

3. The DOJ Panel precisely allowed the parties to adduce more evidence in their behalf and for the panel to study the evidence

submitted more fully.

4. Petitioner’s argument lacks appeal for it lies on the faulty assumption that the decision whom to prosecute is a judicial function, the

sole prerogative of the courts and beyond executive and legislative interference. In truth, the prosecution of crimes appertains to

the executive department of government whose principal power and responsibility is to see that our laws are faithfully executed. A

necessary component of this power is the right to prosecute their violators (See R.A. No. 6981 and section 9 of Rule 119 for legal

basis).

With regard to the inconsistencies of the sworn statements of Jessica Alfaro, the Court believes that these have been sufficiently

explained and there is no showing that the inconsistencies were deliberately made to distort the truth.

With regard to the petitioners’ complaint about the prejudicial publicity that attended their preliminary investigation, the Court finds

nothing in the records that will prove that the tone and content of the publicity that attended the investigation of petitioners fatally

Page 14: Article 7 Compiled Digested Cases

infected the fairness and impartiality of the DOJ Panel. Petitioners cannot just rely on the subliminal effects of publicity on the sense of

fairness of the DOJ Panel, for these are basically unbeknown and beyond knowing.

BIRAOGO VS PTC Leave a comment

G.R. No. 192935 December 7, 2010

LOUIS “BAROK” C. BIRAOGO

vs.

THE PHILIPPINE TRUTH COMMISSION OF 2010

x – - – - – - – - – - – - – - – - – - – - – - -x

G.R. No. 193036

REP. EDCEL C. LAGMAN, REP. RODOLFO B. ALBANO, JR., REP. SIMEON A. DATUMANONG, and REP. ORLANDO B. FUA, SR.

vs.

EXECUTIVE SECRETARY PAQUITO N. OCHOA, JR. and DEPARTMENT OF BUDGET AND MANAGEMENT SECRETARY FLORENCIO B. ABAD

FACTS:

Pres. Aquino signed E. O. No. 1 establishing Philippine Truth Commission of 2010 (PTC) dated July 30, 2010.

PTC is a mere ad hoc body formed under the Office of the President with the primary task to investigate reports of graft and corruption committed by third-level public officers and employees, their co-principals, accomplices and accessories during the previous administration, and to submit its finding and recommendations to the President, Congress and the Ombudsman. PTC has all the powers of an investigative body. But it is not a quasi-judicial body as it cannot adjudicate, arbitrate, resolve, settle, or render awards in disputes between contending parties. All it can do is gather, collect and assess evidence of graft and corruption and make recommendations. It may have subpoena powers but it has no power to cite people in contempt, much less order their arrest. Although it is a fact-finding body, it cannot determine from such facts if probable cause exists as to warrant the filing of an information in our courts of law.

Petitioners asked the Court to declare it unconstitutional and to enjoin the PTC from performing its functions. They argued that:

(a) E.O. No. 1 violates separation of powers as it arrogates the power of the Congress to create a public office and appropriate funds for its operation.

(b) The provision of Book III, Chapter 10, Section 31 of the Administrative Code of 1987 cannot legitimize E.O. No. 1 because the delegated authority of the President to structurally reorganize the Office of the President to achieve economy, simplicity and efficiency does not include the power to create an entirely new public office which was hitherto inexistent like the “Truth Commission.”

(c) E.O. No. 1 illegally amended the Constitution and statutes when it vested the “Truth Commission” with quasi-judicial powers duplicating, if not superseding, those of the Office of the Ombudsman created under the 1987 Constitution and the DOJ created under the Administrative Code of 1987.

(d) E.O. No. 1 violates the equal protection clause as it selectively targets for investigation and prosecution officials and personnel of the previous administration as if corruption is their peculiar species even as it excludes those of the other administrations, past and present, who may be indictable.

Page 15: Article 7 Compiled Digested Cases

Respondents, through OSG, questioned the legal standing of petitioners and argued that:

1] E.O. No. 1 does not arrogate the powers of Congress because the President’s executive power and power of control necessarily include the inherent power to conduct investigations to ensure that laws are faithfully executed and that, in any event, the Constitution, Revised Administrative Code of 1987, PD No. 141616 (as amended), R.A. No. 9970 and settled jurisprudence, authorize the President to create or form such bodies.

2] E.O. No. 1 does not usurp the power of Congress to appropriate funds because there is no appropriation but a mere allocation of funds already appropriated by Congress.

3] The Truth Commission does not duplicate or supersede the functions of the Ombudsman and the DOJ, because it is a fact-finding body and not a quasi-judicial body and its functions do not duplicate, supplant or erode the latter’s jurisdiction.

4] The Truth Commission does not violate the equal protection clause because it was validly created for laudable purposes.

ISSUES:

1. WON the petitioners have legal standing to file the petitions and question E. O. No. 1;

2. WON E. O. No. 1 violates the principle of separation of powers by usurping the powers of Congress to create and to appropriate funds for public offices, agencies and commissions;

3. WON E. O. No. 1 supplants the powers of the Ombudsman and the DOJ;

4. WON E. O. No. 1 violates the equal protection clause.

RULING:

The power of judicial review is subject to limitations, to wit: (1) there must be an actual case or controversy calling for the exercise of judicial power; (2) the person challenging the act must have the standing to question the validity of the subject act or issuance; otherwise stated, he must have a personal and substantial interest in the case such that he has sustained, or will sustain, direct injury as a result of its enforcement; (3) the question of constitutionality must be raised at the earliest opportunity; and (4) the issue of constitutionality must be the very lis mota of the case.

1. The petition primarily invokes usurpation of the power of the Congress as a body to which they belong as members. To the extent the powers of Congress are impaired, so is the power of each member thereof, since his office confers a right to participate in the exercise of the powers of that institution.

Legislators have a legal standing to see to it that the prerogative, powers and privileges vested by the Constitution in their office remain inviolate. Thus, they are allowed to question the validity of any official action which, to their mind, infringes on their prerogatives as legislators.

With regard to Biraogo, he has not shown that he sustained, or is in danger of sustaining, any personal and direct injury attributable to the implementation of E. O. No. 1.

Locus standi is “a right of appearance in a court of justice on a given question.” In private suits, standing is governed by the “real-parties-in interest” rule. It provides that “every action must be prosecuted or defended in the name of the real party in interest.” Real-party-in interest is “the party who stands to be benefited or injured by the judgment in the suit or the party entitled to the avails of the suit.”

Difficulty of determining locus standi arises in public suits. Here, the plaintiff who asserts a “public right” in assailing an allegedly illegal official action, does so as a representative of the general public. He has to show that he is entitled to seek judicial protection. He has to make out a sufficient interest in the vindication of the public order and the securing of relief as a “citizen” or “taxpayer.

Page 16: Article 7 Compiled Digested Cases

The person who impugns the validity of a statute must have “a personal and substantial interest in the case such that he has sustained, or will sustain direct injury as a result.” The Court, however, finds reason in Biraogo’s assertion that the petition covers matters of transcendental importance to justify the exercise of jurisdiction by the Court. There are constitutional issues in the petition which deserve the attention of this Court in view of their seriousness, novelty and weight as precedents

The Executive is given much leeway in ensuring that our laws are faithfully executed. The powers of the President are not limited to those specific powers under the Constitution. One of the recognized powers of the President granted pursuant to this constitutionally-mandated duty is the power to create ad hoc committees. This flows from the obvious need to ascertain facts and determine if laws have been faithfully executed. The purpose of allowing ad hoc investigating bodies to exist is to allow an inquiry into matters which the President is entitled to know so that he can be properly advised and guided in the performance of his duties relative to the execution and enforcement of the laws of the land.

2. There will be no appropriation but only an allotment or allocations of existing funds already appropriated. There is no usurpation on the part of the Executive of the power of Congress to appropriate funds. There is no need to specify the amount to be earmarked for the operation of the commission because, whatever funds the Congress has provided for the Office of the President will be the very source of the funds for the commission. The amount that would be allocated to the PTC shall be subject to existing auditing rules and regulations so there is no impropriety in the funding.

3. PTC will not supplant the Ombudsman or the DOJ or erode their respective powers. If at all, the investigative function of the commission will complement those of the two offices. The function of determining probable cause for the filing of the appropriate complaints before the courts remains to be with the DOJ and the Ombudsman. PTC’s power to investigate is limited to obtaining facts so that it can advise and guide the President in the performance of his duties relative to the execution and enforcement of the laws of the land.

4. Court finds difficulty in upholding the constitutionality of Executive Order No. 1 in view of its apparent transgression of the equal protection clause enshrined in Section 1, Article III (Bill of Rights) of the 1987 Constitution.

Equal protection requires that all persons or things similarly situated should be treated alike, both as to rights conferred and responsibilities imposed. It requires public bodies and institutions to treat similarly situated individuals in a similar manner. The purpose of the equal protection clause is to secure every person within a state’s jurisdiction against intentional and arbitrary discrimination, whether occasioned by the express terms of a statue or by its improper execution through the state’s duly constituted authorities.

There must be equality among equals as determined according to a valid classification. Equal protection clause permits classification. Such classification, however, to be valid must pass the test of reasonableness. The test has four requisites: (1) The classification rests on substantial distinctions; (2) It is germane to the purpose of the law; (3) It is not limited to existing conditions only; and (4) It applies equally to all members of the same class.

The classification will be regarded as invalid if all the members of the class are not similarly treated, both as to rights conferred and obligations imposed.

Executive Order No. 1 should be struck down as violative of the equal protection clause. The clear mandate of truth commission is to investigate and find out the truth concerning the reported cases of graft and corruption during the previous administration only. The intent to single out the previous administration is plain, patent and manifest.

Arroyo administration is but just a member of a class, that is, a class of past administrations. It is not a class of its own. Not to include past administrations similarly situated constitutes arbitrariness which the equal protection clause cannot sanction. Such discriminating differentiation clearly reverberates to label the commission as a vehicle for vindictiveness and selective retribution. Superficial differences do not make for a valid classification.

The PTC must not exclude the other past administrations. The PTC must, at least, have the authority to investigate all past administrations.

Page 17: Article 7 Compiled Digested Cases

The Constitution is the fundamental and paramount law of the nation to which all other laws must conform and in accordance with which all private rights determined and all public authority administered. Laws that do not conform to the Constitution should be stricken down for being unconstitutional.

WHEREFORE, the petitions are GRANTED. Executive Order No. 1 is hereby declared UNCONSTITUTIONAL insofar as it is violative of the equal protection clause of the Constitution.

SECTION 2

G.R. No. 161434 March 3, 2004

MARIA JEANETTE C. TECSON and FELIX B. DESIDERIO, JR. vs.COMELEC, FPJ and VICTORINO X. FORNIER,

G.R. No. 161634 March 3, 2004

ZOILO ANTONIO VELEZ vs.FPJ

G. R. No. 161824 March 3, 2004

VICTORINO X. FORNIER, vs. HON. COMMISSION ON ELECTIONS and FPJ

Facts:

Petitioners sought for respondent Poe’s disqualification in the presidential elections for having allegedly misrepresented material facts in his (Poe’s) certificate of candidacy by claiming that he is a natural Filipino citizen despite his parents both being foreigners. Comelec dismissed the petition, holding that Poe was a Filipino Citizen. Petitioners assail the jurisdiction of the Comelec, contending that only the Supreme Court may resolve the basic issue on the case under Article VII, Section 4, paragraph 7, of the 1987 Constitution.

Issue:

Whether or not it is the Supreme Court which had jurisdiction.

Whether or not Comelec committed grave abuse of discretion in holding that Poe was a Filipino citizen.

Ruling:

1.) The Supreme Court had no jurisdiction on questions regarding “qualification of a candidate” for the presidency or vice-presidency before the elections are held.

"Rules of the Presidential Electoral Tribunal" in connection with Section 4, paragraph 7, of the 1987 Constitution, refers to “contests” relating to the election, returns and qualifications of the "President" or "Vice-President", of the Philippines which the Supreme Court may take cognizance, and not of "candidates" for President or Vice-President before the elections.

2.) Comelec committed no grave abuse of discretion in holding Poe as a Filipino Citizen.

The 1935 Constitution on Citizenship, the prevailing fundamental law on respondent’s birth, provided that among the citizens of the Philippines are "those whose fathers are citizens of the Philippines."

Tracing respondent’s paternal lineage, his grandfather Lorenzo, as evidenced by the latter’s death certificate was identified as a Filipino Citizen. His citizenship was also drawn from the presumption that having died in 1954 at the age of 84, Lorenzo would have been born in 1980. In the absence of any other evidence, Lorenzo’s place of residence upon his death in 1954 was presumed to be the place of residence prior his death, such that Lorenzo Pou would have benefited from the "en masse Filipinization" that the Philippine Bill had effected in 1902. Being so, Lorenzo’s citizenship would have extended to his son, Allan---respondent’s father.

Respondent, having been acknowledged as Allan’s son to Bessie, though an American citizen, was a Filipino citizen by virtue of paternal filiation as evidenced by the respondent’s birth certificate. The 1935 Constitution on citizenship did not make a distinction on the legitimacy or illegitimacy of

Page 18: Article 7 Compiled Digested Cases

the child, thus, the allegation of bigamous marriage and the allegation that respondent was born only before the assailed marriage had no bearing on respondent’s citizenship in view of the established paternal filiation evidenced by the public documents presented.

But while the totality of the evidence may not establish conclusively that respondent FPJ is a natural-born citizen of the Philippines, the evidence on hand still would preponderate in his favor enough to hold that he cannot be held guilty of having made a material misrepresentation in his certificate of candidacy in violation of Section 78, in relation to Section 74 of the Omnibus Election Code.

SECTION 4

MACALINTAL V COMELEC

Political Law – Election Laws – Absentee Voters Act – Proclamation of Winners in a National Elections

Romulo Macalintal, as a lawyer and a taxpayer, questions the validity of the Overseas Absentee Voting Act of 2003 (R.A. 9189). He

questions the validity of the said act on the following grounds, among others:

1. That the provision that a Filipino already considered an immigrant abroad can be allowed to participate in absentee voting provided he

executes an affidavit stating his intent to return to the Philippines is void because it dispenses of the requirement that a voter must be a

resident of the Philippines for at least one year and in the place where he intends to vote for at least 6 months immediately preceding

the election;

2. That the provision allowing the Commission on Elections (COMELEC) to proclaim winning candidates insofar as it affects the canvass

of votes and proclamation of winning candidates for president and vice-president, is unconstitutional because it violates the

Constitution for it is Congress which is empowered to do so.

ISSUE: Whether or not Macalintal’s arguments are correct.

HELD: No.

1. There can be no absentee voting if the absentee voters are required to physically reside in the Philippines within the period required for

non-absentee voters. Further, as understood in election laws, domicile and resident are interchangeably used. Hence, one is a resident

of his domicile (insofar as election laws is concerned). The domicile is the place where one has the intention to return to. Thus, an

immigrant who executes an affidavit stating his intent to return to the Philippines is considered a resident of the Philippines for

purposes of being qualified as a voter (absentee voter to be exact). If the immigrant does not execute the affidavit then he is not

qualified as an absentee voter.

2. The said provision should be harmonized. It could not be the intention of Congress to allow COMELEC to include the proclamation of

the winners in the vice-presidential and presidential race. To interpret it that way would mean that Congress allowed COMELEC to

usurp its power. The canvassing and proclamation of the presidential and vice presidential elections is still lodged in Congress and was

in no way transferred to the COMELEC by virtue of RA 9189.

AQUILINO Q. PIMENTEL, JR.

versus

JOINT COMMITTEE OF CONGRESS TO CANVASS THE VOTES FOR PRESIDENT & VICE PRESIDENT IN THE MAY 10 2004 ELECTIONS

Facts:

By a petition for prohibition, Senator Aquilino Q. Pimentel, Jr. seeks a judgment declaring null and void the continued existence of the Joint Committee of Congress to determine the authenticity and due execution of the certificates of canvass and preliminarily canvass the votes cast for Presidential and Vice Presidential candidates in the May 10 2004 elections following the adjournment of Congress on June 11 2004.

The petition corollarily prays for the issuance of a writ of prohibition directing the Joint Committee to cease and desist from conducting any further proceedings pursuant to the Rules of the Joint Public Session of Congress on Canvassing.

Issue:

Whether or not legislative procedure, precedent or practice as borne out by the rules of both Houses of Congress supports Pimentel’s arguments against the existence and proceedings of the Joint Committee of Congress after the adjournment of Congress.

Held:

Page 19: Article 7 Compiled Digested Cases

NO. Pimentel’s claim that his arguments are buttressed by “legislative procedure, precedent or practice as borne out by the rules of both Houses of Congress” is directly contradicted by Section 42 of Rule XIV of the Rules adopted by the Senate, of which he is an incumbent member.

Moreover, the precedents set by the 1992 and 1998 Presidential Elections do not support the move to stop the ongoing canvassing by the Joint Committee. Thus, during the 1992 Presidential elections, both Houses of Congress adjourned on 25 May 1992. Thereafter, on 22 June 1992, the Eight Congress convened in joint public session as the National Board of Canvassers, and on even date proclaimed Fidel V. Ramos and Joseph Ejercito Estrada as President and Vice President, respectively.

Macalintal vs PET, GR 191618, June 7, 2011

(Admin Law, PET, Quasi-judicial power)

Facts: Par 7, Sec 4, Art VII of the 1987 Constitution provides: “The Supreme Court, sitting en banc, shall be the sole judge of all contests relating to the election, returns, and qualifications of the President or Vice-President, and may promulgate its rules for the purpose.”

Sec 12, Art. VIII of the Constitution provides: The Members of the Supreme Court and of other courts established by law shall not be designated to any agency performing quasi-judicial or administrative functions.

The case at bar is a motion for reconsideration filed by petitioner of the SC’s decision dismissing the former’s petition and declaring the establishment of the respondent PET as constitutional.

Petitioner argues that PET is unconstitutional on the ground that Sec 4, Art VII of the Constitution does not provide for the creation of the PET, and it violates Sec 12, Art VIII of the Constitution.

The Solicitor General maintains that the constitution of the PET is on firm footing on the basis of the grant of authority to the Supreme Court to be the sole judge of all election contests for the President or Vice-President under par 7, Sec 4, Art VII of the Constitution.

Issue:

Whether or not PET is constitutional.

Whether or not PET exercises quasi-judicial power.

Held:

Yes. The explicit reference of the Members of the Constitutional Commission to a Presidential Electoral Tribunal, with Fr. Joaquin Bernas categorically declaring that in crafting the last paragraph of Sec. 4, Art VII of the 1987 Constitution, they “constitutionalized what was statutory.” Judicial power granted to the Supreme Court by the same Constitution is plenary. And under the doctrine of necessary implication, the additional jurisdiction bestowed by the last paragraph of Section 4, Article VII of the Constitution to decide presidential and vice-presidential elections contests includes the means necessary to carry it into effect.

No. The traditional grant of judicial power is found in Section 1, Article VIII of the Constitution which provides that the power “shall be vested in one Supreme Court and in such lower courts as may be established by law.” The set up embodied in the Constitution and statutes characterize the resolution of electoral contests as essentially an exercise of judicial power. When the Supreme Court, as PET, resolves a presidential or vice-presidential election contest, it performs what is essentially a judicial power.

The COMELEC, HRET and SET are not, strictly and literally speaking, courts of law. Although not courts of law, they are, nonetheless, empowered to resolve election contests which involve, in essence, an exercise of judicial power, because of the explicit constitutional empowerment found in Section 2(2), Article IX-C (for the COMELEC) and Section 17, Article VI (for the Senate and House Electoral Tribunals) of the Constitution.

Page 20: Article 7 Compiled Digested Cases

POE, Jr. vs. Arroyo, PET case no. 0002, March 29, 2005

FACTS: Fernando Poe Jr. (FPJ) filed an election protest at the Presidential Electoral Tribunal (PET) against the proclaimed winner of the 2004 presidential elections. During the pendency of the case, FPJ died. His widow, Susan Roces, claimed before the PET that there was an urgent need for her to substitute her husband in the election protest that he had filed as it is of paramount interest to the Filipino people.

ISSUE: May the widow substitute/intervene for the protestant who died during the pendency of the latter’s presidential protest case?

RULING: No. The fundamental rule applicable in a presidential election protest is Rule 14 of the PET Rules. It provides that only the registered candidate for President or Vice President of the Philippines who has received the second or third highest number of votes may timely contest the election of the proclaimed winner. Furthermore, a public office is personal to the public officer and is not a property capable of being transmitted to his heirs upon his death. A real party in interest is the party who would be benefited or injured by the judgment, and the party who is entitled to the avails of the suit. Thus, Susan Roces, is not a real-party-in-interest to the election protest of her husband FPJ.

LOREN B. LEGARDA v. NOLI L. DE CASTRO

P.E.T. Case No. 003, 18 January 2008, Presidential Electoral Tribunal, (Quisumbing, J. )

We are also in agreement that the protestant, in assuming the office of Senator and discharging her duties as such, which fact we can take judicial notice of, has effectively abandoned or withdrawn her protest, or abandoned her determination to protect and pursue the public interest involved in the matter of who is the real choice of the electorate.

FACTS OF THE CASE:

Petitioner Loren B. Legarda filed before the Presidential Electoral Tribunal a petition to annul the proclamation of Respodent Noli L. De Castro as the Vice-President of the Philippines. The protest filed by Legarda consisted of two aspects. The First Aspect covers the alleged erroneous, manipulated and/or falsified results of the election. While the Second pertains to the revision of the ballots of the precincts specified in the protest. The Second Aspect was earlier dismissed by the Supreme Court for the failure of Legarda to pay the required deposit for the expenses.

ISSUE:

Whether or not petitioner clearly and convincingly proved the presence of manipulation or falsification of election results

HELD:

Petition DISMISSED.

We are also in agreement that the protestant, in assuming the office of Senator and discharging her duties as such, which fact we can take judicial notice of, has effectively abandoned or withdrawn her protest, or abandoned her determination to protect and pursue the public interest involved in the matter of who is the real choice of the electorate. The most relevant precedent on this issue is Defensor-Santiago v. Ramos, a decision rendered by this Tribunal, which held that:

The term of office of the Senators elected in the 8 May 1995 election is six years, the first three of which coincides with the last three years of the term of the President elected in the 11 May 1992 synchronized elections. The latter would be Protestant Santiago’s term if she would succeed in proving in the instant protest that she was the true winner in the 1992 elections. In assuming the office of Senator then, the Protestant has effectively abandoned or withdrawn this protest, or at the very least, in the language of Moraleja, abandoned her “determination to protect and pursue the public interest involved in the matter of who is the real choice of the electorate.” Such abandonment or withdrawal operates to render moot the instant protest. Moreover, the dismissal of this protest would serve public interest as it would dissipate the aura of uncertainty as to the

Page 21: Article 7 Compiled Digested Cases

results of the 1992 presidential election, thereby enhancing the all-[too] crucial political stability of the nation during this period of national recovery.

It must also be stressed that under the Rules of the Presidential Electoral Tribunal, an election protest may be summarily dismissed, regardless of the public policy and public interest implications thereof, on the following grounds:

(1) The petition is insufficient in form and substance;

(2) The petition is filed beyond the periods provided in Rules 14 and 15 hereof;

(3) The filing fee is not paid within the periods provided for in these Rules;

(4) The cash deposit, or the first P100,000.00 thereof, is not paid within 10 days after the filing of the protest; and

(5) The petition or copies thereof and the annexes thereto filed with the Tribunal are not clearly legible.

Other grounds for a motion to dismiss, e.g., those provided in the Rules of Court which apply in a suppletory character, may likewise be pleaded as affirmative defenses in the answer. After which, the Tribunal may, in its discretion, hold a preliminary hearing on such grounds. In sum, if an election protest may be dismissed on technical grounds, then it must be, for a decidedly stronger reason, if it has become moot due to its abandonment by the Protestant.

In the case at bar, protestant’s tenure in the Senate coincides with the term of the Vice-Presidency 2004-2010, that is the subject of her protest.

On the matter of the alleged spurious ER copies, we agree with the protestee that the protestant had not adequately and convincingly rebutted the presumption that as public documents, the Congress-retrieved ER copies, used for the proclamation of the protestee by the NBC, are authentic and duly executed in the regular course of official business. The evidence adduced by protestee to show that the supposed security features and markings in the Congress-retrieved ERs and the COMELEC/NAMFREL’s copies are different, did not categorically establish that the Congress-retrieved ERs are fake and spurious. To overcome the presumption of regularity, there must be evidence that is clear, convincing and more than merely preponderant. Absent such convincing evidence, the presumption must be upheld. In fact, the records show that even the witnesses presented by the protestant testified that they were able to discern security features and markings in the Congress-retrieved ERs. The records also show that witnesses were not made to examine all Congress-retrieved ERs in making observations relative to security features and markings, but only a sample set thereof was utilized, resulting in grave insufficiency in the evidence presented by protestant.

As to the alleged break-in in Congress, which allegedly facilitated the switching of ERs, no conclusive evidence has been given. One of the protestant’s own witnesses, Atty. Artemio Adasa, Deputy General for Legislative Operations of the House of Representatives, categorically denied that a break-in and a switching of ERs had occurred in Congress.

At any rate, as pointed out by protestee, even assuming arguendo that all the votes in the 497 precincts included in the pilot areas for the First Aspect with approximately 99,400 votes are considered in favor of protestant, still the protestant would not be able to overcome the lead of the protestee. The margin in favor of protestee adds up to a total of 881,722 votes, and it would take much more than a hundred thousand votes to overcome this lead. This is what the protestant had set out to do in her protest before the Tribunal, but unfortunately she failed to make out her case. In fact, Taraka and Balindong, the only two municipalities on which protestant anchors her arguments for the First Aspect, would only yield an additional 9,931 votes (4,912 votes for Taraka and 5,019 votes for Balindong), a mere fraction of the lead of protestee over protestant. To say that she could have shown that such fraudulent machination was replicated in several other municipalities of Lanao del Sur and other provinces, such as Basilan, Sulu, Tawi-Tawi, Maguindanao, Sultan Kudarat and Lanao del Sur if she had enough time, is mere conjecture and can not be considered convincing by this Tribunal. It is the protestant herself who admits that she was able to adduce evidence only in Taraka and Balindong, for lack of time. But this Tribunal has been liberal in granting her plea for time extension. To say that the protestant had shown enough evidence to prove that the whole or even half (440,862) of the lead of the protestee over the protestant is spurious, would go against the grain of the evidence on hand. One cannot say that half a million votes were illegally obtained based on unclear evidence of cheating in less than ten thousand. The protestant has been afforded ample opportunity to adduce evidence in her behalf for the First Aspect of the protest but the evidence presented is simply insufficient to convince the Tribunal to render invalid all or even half of the 881,722 votes that protestee had over her in the last elections for Vice-President.

DEFENSOR-SANTIAGO V. RAMOS

P.E.T. Case No. 001 February 13, 1996

FACTS:

The presidential election of 1992 was clouded with much uncertainty as to who is the real winner. However, Congress sitting as Board of Canvassers proclaimed Fidel V. Ramos as duly elected President of the Republic. Protestant filed before the Presidential Electoral Tribunal (PET) for annulment of proclamation on grounds of massive fraud and electoral sabotage among others. While the election contest is still pending, Miriam Defensor-Santiago was elected Senator of the Republic in the mid-term election in 1995.

Page 22: Article 7 Compiled Digested Cases

ISSUE:

WON by assuming the position of a Senator, did Defensor-Santiago have effectively abandoned her Presidential Election protest?

HELD:

YES. An election contest involves a public office in which the public has an interest. In the case at bar, when protestant entered into a political contract with the electorate as Senator, she impliedly waives her vested right to the election contest. More so, corollary to her position is the discharge of her functions. In assuming the office of the Senator then, the Protestant has effectively abandoned or withdrawn the protest, or at very least, abandoned her determination to protect or pursue the public interest involved in the matter of who is the real choice of the electorate. Such abandonment or withdrawal operates to render moot and academic the instant case.

Moreover, the resolution of this protest would serve public interest as it would dissipate the aura of uncertainty as to the results of the 1992 presidential elections. Petition was DISMISSED.

SECTION 8 AND SECTION 11

Estrada vs Desierto G.R. No. 146710-15; Estrada vs Arroyo G.R. No. 146738,  March 2 2001

[Immunity from Suit; Resignation of the President; Justiciable controversy]

FACTS:

It began in October 2000 when allegations of wrong doings involving bribe-taking, illegal gambling, and other forms of corruption were

made against Estrada before the Senate Blue Ribbon Committee. On November 13, 2000, Estrada was impeached by the Hor and, on

December 7, impeachment proceedings were begun in the Senate during which more serious allegations of graft and corruption

against Estrada  were made and were only stopped on January 16, 2001 when 11 senators, sympathetic to the President, succeeded

in suppressing damaging evidence against Estrada. As a result, the impeachment trial was thrown into an uproar as the entire

prosecution panel walked out and Senate President Pimentel resigned after casting his vote against Estrada.

On January 19, PNP and the AFP also withdrew their support for Estrada and joined the crowd at EDSA Shrine. Estrada called for a

snap presidential election to be held concurrently with congressional and local elections on May 14, 2001. He added that he will not run

in this election. On January 20, SC declared that the seat of presidency was vacant, saying that Estrada “constructively resigned his

post”. At noon, Arroyo took her oath of office in the presence of the crowd at EDSA as the 14th President. Estrada and his family later

left Malacañang Palace. Erap, after his fall, filed petition for prohibition with prayer for WPI. It sought to enjoin the respondent

Ombudsman from “conducting any further proceedings in cases filed against him not until his term as president ends. He also prayed

for judgment “confirming Estrada to be the lawful and incumbent President of the Republic of the Philippines temporarily unable to

discharge the duties of his office.

ISSUE(S):

1. WoN the petition presents a justiciable controversy.

2. WoN Estrada resigned as President.

3. WoN Arroyo is only an acting President.

4. WoN the President enjoys immunity from suit.

5. WoN the prosecution of Estrada should be enjoined due to prejudicial publicity.

RULING:

1. Political questions-  "to those questions which, under the Constitution, are to be decided by the people in their sovereign capacity, or

in regard to which full discretionary authority has been delegated to the legislative or executive branch of the government. It is

concerned with issues dependent upon the wisdom, not legality of a particular measure."

Legal distinction between EDSA People Power I EDSA People Power II:

EDSA I EDSA II

exercise of the people power of

revolution which overthrew the

whole government.

exercise of people power of

freedom of speech and freedom of

assemblyto petition the government

Page 23: Article 7 Compiled Digested Cases

for redress of grievances which only

affected the office of the President.

extra constitutional and the

legitimacy of the new government

that resulted from it cannot be the

subject of judicial review

intra constitutional and the

resignation of the sitting President

that it caused and the succession of

the Vice President as President are

subject to judicial review.

presented a political question; involves legal questions.

The cases at bar pose legal and not political questions. The principal issues for resolution require the proper interpretation of certain

provisions in the 1987 Constitution: Sec 1 of Art II, and Sec 8 of Art VII, and the allocation of governmental powers under Sec 11 of Art

VII. The issues likewise call for a ruling on the scope of presidential immunity from suit. They also involve the correct calibration of the

right of petitioner against prejudicial publicity.

2. Elements of valid resignation: (a)an intent to resign and (b) acts of relinquishment. Both were present when President Estrada left the

Palace.

Totality of prior contemporaneous posterior facts and circumstantial evidence— bearing material relevant issues—President Estrada is

deemed to have resigned— constructive resignation.

SC declared that the resignation of President Estrada could not be doubted as confirmed by his leaving Malacañan Palace. In the press

release containing his final statement:

1. He acknowledged the oath-taking of the respondent as President;

2. He emphasized he was leaving the Palace for the sake of peace and in order to begin the healing process (he did not say that he

was leaving due to any kind of disability and that he was going to reassume the Presidency as soon as the disability disappears);

3. He expressed his gratitude to the people for the opportunity to serve them as President (without doubt referring to the past

opportunity);

4. He assured that he will not shirk from any future challenge that may come in the same service of the country;

5. He called on his supporters to join him in promotion of a constructive national spirit of reconciliation and solidarity.

Intent to resign—must be accompanied by act of relinquishment—act or omission before, during and after January 20, 2001.

3. The Congress passed House Resolution No. 176 expressly stating its support to Gloria Macapagal-Arroyo as President of the

Republic of the Philippines and subsequently passed H.R. 178 confirms the nomination of Teofisto T. Guingona Jr. As Vice President.

Senate passed  HR No. 83 declaring the Impeachment Courts as Functius Officio and has been terminated. It is clear is that both

houses of Congress recognized Arroyo as the President. Implicitly clear in that recognition is the premise that the inability of Estrada is

no longer temporary as the Congress has clearly rejected his claim of inability.

The Court therefore cannot exercise its judicial power for this is political in nature and addressed solely to Congress by constitutional

fiat.  In fine, even if Estrada can prove that he did not resign, still, he cannot successfully claim that he is a President on leave on the

ground that he is merely unable to govern temporarily. That claim has been laid to rest by Congress and the decision that Arroyo is the

de jure, president made by a co-equal branch of government cannot be reviewed by this Court.

4. The cases filed against Estrada are criminal in character. They involve plunder, bribery and graft and corruption. By no stretch of the

imagination can these crimes, especially plunder which carries the death penalty, be covered by the alleged mantle of immunity of a

non-sitting president. He cannot cite any decision of this Court licensing the President to commit criminal acts and wrapping him with

post-tenure immunity from liability. The rule is that unlawful acts of public officials are not acts of the State and the officer who acts

illegally is not acting as such but stands in the same footing as any trespasser.

5. No. Case law will tell us that a right to a fair trial and the free press are incompatible. Also, since our justice system does not use the

jury system, the judge, who is a learned and legally enlightened individual, cannot be easily manipulated by mere publicity. The Court

also said that Estrada did not present enough evidence to show that the publicity given the trial has influenced the judge so as to render

the judge unable to perform. Finally, the Court said that the cases against Estrada were still undergoing preliminary investigation, so the

publicity of the case would really have no permanent effect on the judge and that the prosecutor should be more concerned with justice

and less with prosecution.

SECTION 13

194 SCRA 317 – Political Law – Ex Officio Officials – Members of the Cabinet – Singularity of Office - EO 284

In July 1987, the President Corazon Aquino issued Executive Order No.  284 which allowed members of the Cabinet, their

undersecretaries and assistant secretaries to hold other government offices or positions in addition to their primary positions subject to

limitations set therein. The Civil Liberties Union (CLU) assailed this EO averring that such law is unconstitutional. The constitutionality of

EO 284 is being challenged by CLU on the principal submission that it adds exceptions to Sec 13, Article 7 of the Constitution which

provides:

Page 24: Article 7 Compiled Digested Cases

“Sec. 13. The President, Vice-President, the Members of the Cabinet, and their deputies or assistants shall not, unless otherwise

provided in this Constitution, hold any other office or employment during their tenure. They shall not, during said tenure, directly or

indirectly practice any other profession, participate in any business, or be financially interested in any contract with, or in any franchise,

or special privilege granted by the Government or any subdivision, agency, or instrumentality thereof, including government-owned or

controlled corporations or their subsidiaries. They shall strictly avoid conflict of interest in the conduct of their office.”

CLU avers that by virtue of the phrase “unless otherwise provided in this Constitution“, the only exceptions against holding any other

office or employment in Government are those provided in the Constitution, namely: (i) The Vice-President may be appointed as a

Member of the Cabinet under Sec 3, par. (2), Article 7; and (ii) the Secretary of Justice is an ex-officio member of the Judicial and Bar

Council by virtue of Sec 8 (1), Article 8.

ISSUE: Whether or not EO 284 is constitutional.

HELD: No, it is unconstitutional. It is clear that the 1987 Constitution seeks to prohibit the President, Vice-President, members of the

Cabinet, their deputies or assistants from holding during their tenure multiple offices or employment in the government, except in those

cases specified in the Constitution itself and as above clarified with respect to posts held without additional compensation in an ex-

officio capacity as provided by law and as required by the primary functions of their office, the citation of Cabinet members (then called

Ministers) as examples during the debate and deliberation on the general rule laid down for all appointive officials should be considered

as mere personal opinions which cannot override the constitution’s manifest intent and the people’s understanding thereof.

In the light of the construction given to Sec 13, Art 7 in relation to Sec 7, par. (2), Art IX-B of the 1987 Constitution, EO 284 is

unconstitutional. Ostensibly restricting the number of positions that Cabinet members, undersecretaries or assistant secretaries may

hold in addition to their primary position to not more than 2 positions in the government and government corporations, EO 284 actually

allows them to hold multiple offices or employment in direct contravention of the express mandate of Sec 13, Art 7 of the 1987

Constitution prohibiting them from doing so, unless otherwise provided in the 1987 Constitution itself.

 

G. R. No. 85468, September 07, 1989

DOROMA VS. SANDIGANBAYAN, Ombudsman and Special Prosecutor

FACTS:

Quintin S. Doromal, a former Commissioner of the Presidential Commission on Good Government (PCGG), for violation of the Anti-Graft and Corrupt Practices Act (RA 3019), Sec. 3(h), in connection with his shareholdings and position as president and director of the Doromal International Trading Corporation (DITC) which submitted bids to supply P61 million worth of electronic, electrical, automotive, mechanical and airconditioning equipment to the Department of Education, Culture and Sports (or DECS) and the National Manpower and Youth Council (or NMYC).

An information was then filed by the “Tanodbayan” against Doromal for the said violation and a preliminary investigation was conducted.

The petitioner then filed a petition for certiorari and prohibition questioning the jurisdiction of the “Tanodbayan” to file the information without the approval of the Ombudsman.

The Supreme Court held that the incumbent Tanodbayan (called Special Prosecutor under the 1987 Constitution and who is supposed to retain powers and duties NOT GIVEN to the Ombudsman) is clearly without authority to conduct preliminary investigations and to direct the filing of criminal cases with the Sandiganbayan, except upon orders of the Ombudsman. Subsequently annulling the information filed by the “Tanodbayan”.

A new information, duly approved by the Ombudsman, was filed in the Sandiganbayan, alleging that the Doromal, a public officer, being then a Commissioner of the Presidential Commission on Good Government, did then and there wilfully and unlawfully, participate in a business through the Doromal International Trading Corporation, a family corporation of which he is the President, and which company participated in the biddings conducted by the Department of Education, Culture and Sports and the National Manpower & Youth Council, which act or participation is prohibited by law and the constitution.

The petitioner filed a motion to quash the information on the ground that it was invalid since there had been no preliminary investigation for the new information that was filed against him.

The motion was denied by Sandiganbayan claiming that another preliminary investigation is unnecessary because both old and new informations involve the same subject matter.

ISSUES:

Whether or not the act of Doromal would constitute a violation of the Constitution.

Whether or not preliminary investigation is necessary even if both informations involve the same subject matter.

Page 25: Article 7 Compiled Digested Cases

Whether or not the information shall be effected as invalid due to the absence of preliminary investigation.

HELD:

Yes, as to the first and second issuses. No, as to the third issue. Petition was granted by the Supreme Court.

RATIO:

(1) The presence of a signed document bearing the signature of Doromal as part of the application to bid shows that he can rightfully be charged with having participated in a business which act is absolutely prohibited by Section 13 of Article VII of the Constitution" because "the DITC remained a family corporation in which Doromal has at least an indirect interest."

Section 13, Article VII of the 1987 Constitution provides that "the President, Vice-President, the members of the Cabinet and their deputies or assistants shall not... during (their) tenure, ...directly or indirectly... participate in any business.

(2) The right of the accused to a preliminary investigation is "a substantial one." Its denial over his opposition is a "prejudicial error, in that it subjects the accused to the loss of life, liberty, or property without due process of law" provided by the Constitution.

Since the first information was annulled, the preliminary investigation conducted at that time shall also be considered as void. Due to that fact, a new preliminary investigation must be conducted.

(3) The absence of preliminary investigation does not affect the court's jurisdiction over the case. Nor do they impair the validity of the information or otherwise render it defective; but, if there were no preliminary investigations and the defendants, before entering their plea, invite the attention of the court to their absence, the court, instead of dismissing the information should conduct such investigation, order the fiscal to conduct it or remand the case to the inferior court so that the preliminary investigation may be conducted.

WHEREFORE, the petition for certiorari and prohibition is granted. The Sandiganbayan shall immediately remand Criminal Case No. 12893 to the Office of the Ombudsman for preliminary investigation and shall hold in abeyance the proceedings before it pending the result of such investigation.

Bitonio vs Commission on Audit

GR No. 14732

March 12, 2004

Instant petition filed under Rule 64 of the Revised Rules of Court seeks the annulment of the decision of COA denying the petitioner’s

motion for reconsideration of the COA Notices of Disallowance.

Benedicto Ernesto R. Bitonio, Jr was appointed Director IV of the Bureau of Labor Relations in the DOLE. Acting Secretary Jose S.

Brillantes of DOLE designated Bitonio to be the DOLE representative to the Boiard directors of PEZA. After post audit of PEZA’s

disbursement transactions, COA disallowed the per diem of Bitonio.

Arguments of COA:

1.    Cabinet members, their deputies and assistants holding other offices in addition to primary office are not allowed to receive

compensation to subsequent office

Argument of Bitonio:

1.    Rank equivalent to a Secretary, Undersecretary or Assistant Secretary and other appointive officials below the rank of Assistant

secretary are not covered by the prohibition

2.    Section 11 RA No. 7916 was enacted four years after Civil Liberties Union become final thus Congress is presumed to be aware of the

parameters.

3.    RA No. 7916 is presumed to be valid.

4.    RA No. 7916 is more superior than COA Memorandum No. 97-038. (Statute vs Administrative directive)

Ruling of the Court:

1.    Article VII Section 13 of 1987 Constitution’

2.    Dela Cruz v Commission on Audit – if a secretary of Finance attends a monetary board meeting as an ex officio member, he is actually

and in legal compensation performing the primary function of his principal office

3.    Whatever prohibitions or restrictions the member is subjected, the representative is likewise, not exempted.

4.    Constitution is more superior than a statuteRA No. 7916 was later amended by RA No. 8748 to cure the defect

Public Interest Center vs. ELMA

Page 26: Article 7 Compiled Digested Cases

June 30, 2006, Chico-Nazario

*concurrent appointments, incompatible office

N: CPM + TRO to declare null and void the concurrent appointments of ELMA as PCGG Chair and as Chief Presidential Legal Counsel

F: Elma was appointed as PCGG Chair Oct 1998. Later on he was appointed as CPLC (Jan 1999 during his term), but waived any remuneration that he may receive as CPLC.

Supervening events: There’s actually no more controversy involved: In 2001, Elma was replaced by Sabio as PCGG. Nachura was then appointed as CPLC but pending resolution of the case, he was appointed SOLGEN.

Arguments: Public Interest Center

CLU vs. Exec Sec: Art IX-B, Sec 7, par2 and Art VII, Sec13 are violated by concurrent appointments

CPLC and PCGG Chair are incompatible offices

Arguments: Elma

As interpreted in CLU vs. Exec Sec, the mentioned consti provisions don’t cover other public officials given the rank of Secretary, Undersecretary, or Assistant Secretary.

His appointment falls under the exceptions in Art IX-B, Sec7

The 2 positions are not incompatible

NOTE: even if issue already moot, SC still took cognizance of the case because the case is capable of repetition, and to serve as a guide to the bench.

whether the position of the PCGG Chairman or that of the CPLC falls under the prohibition against multiple offices imposed by Section 7, par. 2, Article IX-B of the 1987 Constitution

YES.

The crucial test in determining whether incompatibility exists between two offices was laid out in People v. Green[13] - whether one office is subordinate to the other, in the sense that one office has the right to interfere with the other.

[I]ncompatibility between two offices, is an inconsistency in the functions of the two; x x x Where one office is not subordinate to the other, nor the relations of the one to the other such as are inconsistent and repugnant, there is not that incompatibility from which the law declares that the acceptance of the one is the vacation of the other. The force of the word, in its application to this matter is, that from the nature and relations to each other, of the two places, they ought not to be held by the same person, from the contrariety and antagonism which would result in the attempt by one person to faithfully and impartially discharge the duties of one, toward the incumbent of the other. x x x The offices must subordinate, one [over] the other, and they must, per se, have the right to interfere, one with the other, before they are incompatible at common law. x x x

***In this case, an incompatibility exists between the positions of the PCGG Chairman and the CPLC. The duties of the CPLC include giving independent and impartial legal advice on the actions of the heads of various executive departments and agencies and to review investigations involving heads of executive departments and agencies, as well as other Presidential appointees. The PCGG is, without question, an agency under the Executive Department. Thus, the actions of the PCGG Chairman are subject to the review of the CPLC.

*note: Memorandum Order No. 152, issued on 9 July 2004 (provides that CPLC review Decision on investigation involving Cabinet Secretaries, agency heads, or Presidential appointees with the rank of Secretary conducted by the Presidential Anti-Graft Commission (PAGC))

whether such appointments violate the other constitutional provision regarding multiple offices, Section 13, Article VII of the 1987 Constitution

NO if based on position. YES if based on primary functions test.

the strict prohibition under Section 13, Article VII of the 1987 Constitution is not applicable to the PCGG Chairman nor to the CPLC, as neither of them is a secretary, undersecretary, nor an assistant secretary, even if the former may have the same rank as the latter positions.

Page 27: Article 7 Compiled Digested Cases

*Review ulit CLU vs. Exec Sec: The language of Section 13, Article VII is a definite and unequivocal negation of the privilege of holding multiple offices or employment.

The Court cautiously allowed only two exceptions to the rule against multiple offices:

(1) those provided for under the Constitution, such as Section 3, Article VII, authorizing the Vice-President to become a member of the Cabinet; or

(2) posts occupied by the Executive officials specified in Section 13, Article VII without additional compensation in an ex-officio capacity as provided by law and as required by the primary functions of said officials’ office.

…The Court further qualified that additional duties must not only be closely related to, but must be required by the official’s primary functions. Moreover, the additional post must be exercised in an ex-officio capacity, which “denotes an act done in an official character, or as a consequence of office, and without any other appointment or authority than that conferred by the office.”[18] Thus, it will not suffice that no additional compensation shall be received by virtue of the second appointment, it is mandatory that the second post is required by the primary functions of the first appointment and is exercised in an ex-officio capacity.

*Even Section 13, Article VII does not sanction this dual appointment. Appointment to the position of PCGG Chairman is not required by the primary functions of the CPLC, and vice versa.

In sum, the prohibition in Section 13, Article VII of the 1987 Constitution does not apply to respondent Elma since neither the PCGG Chairman nor the CPLC is a Cabinet secretary, undersecretary, or assistant secretary. Even if this Court assumes, arguendo, that Section 13, Article VII is applicable to respondent Elma, he still could not be appointed concurrently to the offices of the PCGG Chairman and CPLC because neither office was occupied by him in an ex-officio capacity, and the primary functions of one office do not require an appointment to the other post. Moreover, even if the appointments in question are not covered by Section 13, Article VII of the 1987 Constitution, said appointments are still prohibited under Section 7, Article IX-B, which covers all appointive and elective officials, due to the incompatibility between the primary functions of the offices of the PCGG Chairman and the CPLC.

FLORES V DRILON

FACTS:

The constitutionality of Sec. 13, par. (d), of R.A. 7227, otherwise known as the "Bases Conversion and Development Act of 1992," under which respondent Mayor Richard J. Gordon of Olongapo City was appointed Chairman and Chief Executive Officer of the Subic Bay Metropolitan Authority (SBMA), is challenged with prayer for prohibition, preliminary injunction and temporary restraining order. Said provision provides the President the power to appoint an administrator of the SBMA provided that in the first year of its operation, the Olongapo mayor shall be appointed as chairman and chief of executive of the Subic Authority. Petitioners maintain that such infringes to the constitutional provision of Sec. 7, first par., Art. IX-B, of the Constitution, which states that "no elective official shall be eligible for appointment or designation in any capacity to any public officer or position during his tenure," The petitioners also contend that Congress encroaches upon the discretionary power of the President to appoint.

ISSUE:

Whether or not said provision of the RA 7227 violates the constitutional prescription against appointment or designation of elective officials to other government posts.

RULING:

The court held the Constitution seeks to prevent a public officer to hold multiple functions since they are accorded with a public office that is a full time job to let them function without the distraction of other governmental duties.

The Congress gives the President the appointing authority which it cannot limit by providing the condition that in the first year of the operation the Mayor of Olongapo City shall assume the Chairmanship. The court points out that the appointing authority the congress gives to the President is no power at all as it curtails the right of the President to exercise discretion of whom to appoint by limiting his choice.

Dennis B. Funa vs. Executive Secretary Eduardo R. Ermita, Office of the President,G.R. No. 184740, February 11, 2010.

Page 28: Article 7 Compiled Digested Cases

Post under Political Law, villarama doctrines at Monday, November 28, 2011 Posted by Schizophrenic Mind

Judicial review; requisites. The courts’ power of judicial review, like almost all other powers conferred by the Constitution, is subject to several limitations, namely: (1) there must be an actual case or controversy calling for the exercise of judicial power; (2) the person challenging the act must have “standing” to challenge; he must have a personal and substantial interest in the case, such that he has sustained or will sustain, direct injury as a result of its enforcement; (3) the question of constitutionality must be raised at the earliest possible opportunity; and (4) the issue of constitutionality must be the very lis mota of the case. Respondents assert that the second requisite is absent in this case.

Generally, a party will be allowed to litigate only when (1) he can show that he has personally suffered some actual or threatened injury because of the allegedly illegal conduct of the government; (2) the injury is fairly traceable to the challenged action; and (3) the injury is likely to be redressed by a favorable action. The question on standing is whether such parties have “alleged such a personal stake in the outcome of the controversy as to assure that concrete adverseness which sharpens the presentation of issues upon which the court so largely depends for illumination of difficult constitutional questions.”

In David v. Macapagal-Arroyo, summarizing the rules culled from jurisprudence, the Supreme Court held that taxpayers, voters, concerned citizens, and legislators may be accorded standing to sue, provided that the following requirements are met:

(1) cases involve constitutional issues;

(2) for taxpayers, there must be a claim of illegal disbursement of public funds or that the tax measure is unconstitutional;

(3) for voters, there must be a showing of obvious interest in the validity of the election law in question;

(4) for concerned citizens, there must be a showing that the issues raised are of transcendental importance which must be settled early; and for legislators, there must be a claim that the official action complained of infringes upon their prerogatives as legislators.

Petitioner having alleged a grave violation of the constitutional prohibition against Members of the Cabinet, their deputies and assistants holding two (2) or more positions in government, the fact that he filed this suit as a concerned citizen sufficiently confers him with standing to sue for redress of such illegal act by public officials.

Public officials; multiple office. The prohibition against holding dual or multiple offices or employment under Section 13, Article VII of the 1987 Constitution was held inapplicable to posts occupied by the Executive officials specified therein, without additional compensation in an ex-officio capacity as provided by law and as required by the primary functions of said office. The reason is that these posts do not comprise “any other office” within the contemplation of the constitutional prohibition but are properly an imposition of additional duties and functions on said officials. Apart from their bare assertion that respondent Bautista did not receive any compensation when she was OIC of MARINA, respondents failed to demonstrate clearly that her designation as such OIC was in an ex-officio capacity as required by the primary functions of her office as DOTC Undersecretary for Maritime Transport.

Given the vast responsibilities and scope of administration of the MARINA, we are hardly persuaded by respondents’ submission that respondent Bautista’s designation as OIC of MARINA was merely an imposition of additional duties related to her primary position as DOTC Undersecretary for Maritime Transport. It appears that the DOTC Undersecretary for Maritime Transport is not even a member of the Maritime Industry Board, which includes the DOTC Secretary as Chairman, the MARINA Administrator as Vice-Chairman, and the following as members: Executive Secretary (Office of the President), Philippine Ports Authority General Manager, Department of National Defense Secretary, Development Bank of the Philippines General Manager, and the Department of Trade and Industry Secretary.

It must be stressed though that while the designation was in the nature of an acting and temporary capacity, the words “hold the office” were employed. Such holding of office pertains to both appointment and designation because the appointee or designate performs the duties and functions of the office. The 1987 Constitution in prohibiting dual or multiple offices, as well as incompatible offices, refers to the holding of the office, and not to the nature of the appointment or designation, words which were not even found in Section 13, Article VII nor in Section 7, paragraph 2, Article IX-B. To “hold” an office means to “possess or occupy” the same, or “to be in possession and administration,” which implies nothing less than the actual discharge of the functions and duties of the office.

Page 29: Article 7 Compiled Digested Cases

The disqualification laid down in Section 13, Article VII is aimed at preventing the concentration of powers in the Executive Department officials, specifically the President, Vice-President, Members of the Cabinet and their deputies and assistants. Civil Liberties Union traced the history of the times and the conditions under which the Constitution was framed, and construed the Constitution consistent with the object sought to be accomplished by adoption of such provision, and the evils sought to be avoided or remedied. We recalled the practice, during the Marcos regime, of designating members of the Cabinet, their deputies and assistants as members of the governing bodies or boards of various government agencies and instrumentalities, including government-owned or controlled corporations. This practice of holding multiple offices or positions in the government led to abuses by unscrupulous public officials, who took advantage of this scheme for purposes of self-enrichment. The blatant betrayal of public trust evolved into one of the serious causes of discontent with the Marcos regime. It was therefore quite inevitable and in consonance with the overwhelming sentiment of the people that the 1986 Constitutional Commission would draft into the proposed Constitution the provisions under consideration, which were envisioned to remedy, if not correct, the evils that flow from the holding of multiple governmental offices and employment. Dennis B. Funa vs. Executive Secretary Eduardo R. Ermita, Office of the President,G.R. No. 184740, February 11, 2010.

Section 15

De Rama v. Court of Appeals (Ynares-Santiago, 2001)

Facts:

- Mayor Conrado de Rama won as mayor of Pagbilao, Quezon. One of the first things he did upon assumption of office was to write the Civil Service Commission and seek the recall of the appointments of 14 municipal employees. According to him, said appointments should be recalled as they were “midnight” appointments of the former mayor, Evelyn Abeja. The CSC denied his request saying that the appointments of the 14 employees were made in accordance with law and that the sec.15, art.VII of the Constitution which is being relied upon by Mayor de Rama, pertains only to the appointments of the outgoing President and not of local elective officials.

- Upon appeal to the CA, Mayor de Rama filed a supplemental pleading to the appeal alleging that the appointments were also tainted with fraud since the former mayor did not follow the rule in sec.80 of Ra 7041 that appointments can only be made within 4 months from the publication of the vacancies.

Issue: WON the appointments made by the former mayor should be recalled.

Held and Ratio:

- NO. The CSC has correctly ruled that the appointments were made in accordance with the law. It was already too late for Mayor de Rama to claim that appointments were tainted with fraud since he did not raise this in his first complaint, which only relied on his allegation that the same were midnight appointments. Only the CSC has the power to recall the appointments upon grounds mentioned in the Revised Administrative Code. However, none of the grounds exist to warrant the recall of the said appointments. To grant the mayor’s request is to violate the security of tenure of the appointed employees.

- Aside from this, the Court ruled that it was error for Mayor de Rama to invoke the constitutional prohibition against midnight appointments. According to the Court, this only pertains to appointments made by an outgoing President and is not applicable to appointments made by an outgoing mayor.

Mendoza, dissenting:

- The prohibition against midnight appointments is not limited to those made by an outgoing President. The same covers those made by outgoing elective officials since midnight appointments in general are bad because they are made hurriedly, without due deliberation and careful consideration of the needs of the office and the qualifications of the appointee. Moreover, the offend principle of fairness, justice and righteousness.

CASE DIGEST

ARTURO M. DE CASTRO vs. JUDICIAL AND BAR COUNCIL (JBC) 

G. R. No. 191002. March 17, 2010.

FACTS: 

Page 30: Article 7 Compiled Digested Cases

This case is based on multiple cases field with dealt with the controversy that has arisen from the forthcoming compulsory requirement of Chief Justice Puno on May 17, 2010 or seven days after the presidential election. On December 22, 2009, Congressman Matias V. Defensor, an ex officio member of the JBC, addressed a letter to the JBC, requesting that the process for nominations to the office of the Chief Justice be commenced immediately. In its January 18, 2010 meeting en banc, the JBC passed a resolution which stated that they have unanimously agreed to start the process of filling up the position of Chief Justice to be vacated on May 17, 2010 upon the retirement of the incumbent Chief Justice. As a result, the JBC opened the position of Chief Justice for application or recommendation, and published for that purpose its announcement in the Philippine Daily Inquirer and the Philippine Star. In its meeting of February 8, 2010, the JBC resolved to proceed to the next step of announcing the names of the following candidates to invite to the public to file their sworn complaint, written report, or opposition, if any, not later than February 22, 2010. Although it has already begun the process for the filling of the position of Chief Justice Puno in accordance with its rules, the JBC is not yet decided on when to submit to the President its list of nominees for the position due to the controversy in this case being unresolved. The compiled cases which led to this case and the petitions of intervenors called for either the prohibition of the JBC to pass the shortlist, mandamus for the JBC to pass the shortlist, or that the act of appointing the next Chief Justice by GMA is a midnight appointment. A precedent frequently cited by the parties is the In Re Appointments Dated March 30, 1998 of Hon. Mateo A. Valenzuela and Hon. Placido B. Vallarta as Judges of the RTC of Branch 62, Bago City and of Branch 24, Cabanatuan City, respectively, shortly referred to here as the Valenzuela case, by which the Court held that Section 15, Article VII prohibited the exercise by the President of the power to appoint to judicial positions during the period therein fixed. 

ISSUES: 

1. Whether or not the petitioners have legal standing.

2. Whether or not there is justiciable controversy that is ripe for judicial determination.

3. Whether or not the incumbent President can appoint the next Chief Justice.

4. Whether or not mandamus and prohibition will lie to compel the submission of the shortlist of nominees by the JBC. 

HELD:

1.Petitioners have legal standing because such requirement for this case was waived by the Court. Legal standing is a peculiar concept in constitutional law because in some cases, suits are not brought by parties who have been personally injured by the operation of a law or any other government act but by concerned citizens, taxpayers or voters who actually sue in the public interest.” But even if, strictly speaking, the petitioners “are not covered by the definition, it is still within the wide discretion of the Court to waive the requirement and so remove the impediment to its addressing and resolving the serious constitutional questions raised.” 

2. There is a justiciable issue. The court holds that the petitions set forth an actual case or controversy that is ripe for judicial determination. The reality is that the JBC already commenced the proceedings for the selection of the nominees to be included in a short list to be submitted to the President for consideration of which of them will succeed Chief Justice Puno as the next Chief Justice. Although the position is not yet vacant, the fact that the JBC began the process of nomination pursuant to its rules and practices, although it has yet to decide whether to submit the list of nominees to the incumbent outgoing President or to the next President, makes the situation ripe for judicial determination, because the next steps are the public interview of the candidates, the preparation of the short list of candidates, and the “interview of constitutional experts, as may be needed.” The resolution of the controversy will surely settle – with finality – the nagging questions that are preventing the JBC from moving on with the process that it already began, or that are reasons persuading the JBC to desist from the rest of the process. 

3.Prohibition under section 15, Article VII does not apply to appointments to fill a vacancy in the Supreme Court or to other appointments to the judiciary. The records of the deliberations of the Constitutional Commission reveal that the framers devoted time to meticulously drafting, styling, and arranging the Constitution. Such meticulousness indicates that the organization and arrangement of the provisions of the Constitution were not arbitrarily or whimsically done by the framers, but purposely made to reflect their intention and manifest their vision of what the Constitution should contain. As can be seen, Article VII is devoted to the Executive Department, and, among others, it lists the powers vested by the Constitution in the President. The presidential power of appointment is dealt with in Sections 14, 15 and 16 of the Article. Had the framers intended to extend the prohibition contained in Section 15, Article VII to the appointment of Members of the Supreme Court, they could have explicitly done so. They could not have ignored the meticulous

Page 31: Article 7 Compiled Digested Cases

ordering of the provisions. They would have easily and surely written the prohibition made explicit in Section 15, Article VII as being equally applicable to the appointment of Members of the Supreme Court in Article VIII itself, most likely in Section 4 (1), Article VIII. 

4.Writ of mandamus does not lie against the JBC. Mandamus shall issue when any tribunal, corporation, board, officer or person unlawfully neglects the performance of an act that the law specifically enjoins as a duty resulting from an office, trust, or station. It is proper when the act against which it is directed is one addressed to the discretion of the tribunal or officer. Mandamus is not available to direct the exercise of a judgment or discretion in a particular way.  For mandamus to lie, the following requisites must be complied with: (a) the plaintiff has a clear legal right to the act demanded; (b) it must be the duty of the defendant to perform the act, because it is mandated by law; (c) the defendant unlawfully neglects the performance of the duty enjoined by law; (d) the act to be performed is ministerial, not discretionary; and (e) there is no appeal or any other plain, speedy and adequate remedy in the ordinary course of law.

In re: Valenzuela

In re: Valenzuela

A.M. No. 98-5-01-SC November 9, 1998

Narvasa, C.J.

Facts:

                On March 30, 1998, The President signed appointments of Hon. Mateo Valenzuela and Hon. Placido Vallarta as Judges of

RTC-Bago City and Cabanatuan City, respectively.  These appointments were deliberated, as it seemed to be expressly prohibited by

Art 7 Sec 15 of the Constitution:

Two months immediately before the next presidential elections and up to the end of his term, a President or Acting President shall not

make appointments, except temporary appointments to executive positions when continued vacancies therein will prejudice public

service or endanger public safety.”

A meeting was held on March 9, 1998 by the Judicial and Bar Council to discuss the constitutionality of appointments to the

Court of Appeals (CA) in light of the forthcoming 1998 Presidential elections.  Senior Associate Justice Florenz Regalado, Consultant of

the Council and Member of the 1986 Constitutional Commission, was in the position that “election ban had no application to the CA

based on the Commission’s records”.  This hypothesis was then submitted to the President for consideration together with the

Council’s nominations for 8 vacancies in the CA.

                The Chief Justice (CJ) received on April 6, 1998, an official communication from the Executive Secretary transmitting the

appointments of 8 Associate Justices of CA duly signed on March 11, 1998 (day immediately before the commencement of the ban on

appointments), which implies that the President’s Office did not agree with the hypothesis.

                The President, addressed to the JBC, requested on May 4, 1998 the transmission of the “list of final nominees” for the

vacancy in view of the 90 days imposed by the Constitution (from Feb 13, date present vacancy occurred).   In behalf of the JBC, CJ

sent the reply on May 6 that no session has been scheduled after the May elections for the reason that they apparently did not share

the same view (hypothesis) proposed by the JBC shown by the uniformly dated March 11, 1998 appointments.  However, it appeared

that the Justice Secretary and the other members of the Council took action without waiting for the CJ reply.   This prompted CJ to call

for a meeting on May 7.  On this day, CJ received a letter from the President in reply of the May 6 letter where the President expressed

his view that Article 7 Sec 15 only applied to executive appointments, the whole article being entitled “EXECUTIVE DEPT”.   He posited

that appointments in the Judiciary have special and specific provisions, as follows:

Article 8 Sec 4

“The Supreme Court shall be composed of a Chief Justice and fourteen Associate Justices.  It may sit en banc or in its discretion, in

divisions of three, five, or seven Members.  Any  vacancy shall be filled within ninety days from the occurrence thereof.”

Article 8 Sec 9

“The Members of the Supreme Court and judges in lower courts shall be appointed by the President from the list of at least three

nominees prepared by the Judicial and Bar Council for every vacancy.  Such appointments need no confirmation.

                On May 12, CJ received from Malacañang, the appointments of the 2 Judges of the RTC mentioned.   Considering the

pending proceedings and deliberations on this matter, the Court resolved by refraining the appointees from taking their oaths.  

However, Judge Valenzuela took oath in May 14, 1998 claiming he did so without knowledge on the on-going deliberations.   It should

be noted that the originals of the appointments for both judges had been sent to and received by the CJ on May 12 and is still in the

latter’s office and had not been transmitted yet.  According to Judge Valenzuela, he did so because of the May 7 Malacañang copy of

his appointment.

Page 32: Article 7 Compiled Digested Cases

                In construing Article 7 and 8: when there are no presidential elections, Art. 8 shall apply where vacancies in SC shall be filled

within 90 days otherwise prohibition in Art. 7 must be considered where the President shall not make any appointments.  According to

Fr. Bernas, the reason for prohibition is in order not to tie the hands of the incoming Pres through midnight appointments.

Issue:

                whether, during the period of the ban on appointments imposed by Section 15, Article VII of the, Constitution, the President is

nonetheless required to fill vacancies in the judiciary, in view of Sections 4(1) and 9 of Article VIII; whether he can make appointments

to the judiciary during the period of the ban in the interest of public service.

Held:

                The provisions of the Constitution material to the inquiry at bar read as follows: 3

Sec. 15, Article VII:

Two months immediately before the next presidential elections and up to the end of his term, a President or Acting President shall not

make appointments,except temporary appointments to executive positions when continued vacancies therein will prejudice public

service or endanger public safety.

Sec. 4 (1), Article VIII :

The Supreme Court shall be composed of a Chief Justice and fourteen Associate Justices. It may sit en banc or in its discretion, in

divisions of three, five, or seven Members. Any vacancy shall be filled within ninety days from the occurrence thereof.

Sec. 9, Article VIII :

The members of the Supreme Court and judges in lower courts shall be appointed by the President from a list of at least three

nominees prepared by the Judicial and Bar Council for, every vacancy. Such appointments need no confirmation.

For the lower courts, the President shall issue the appointments within ninety days from the submission of the list.

During the period stated in Section 15. Article VII of the Constitution — “(t)wo months immediately before the next presidential

elections and up to the end his term” — the President is neither required to make appointments to the courts nor allowed to do so; and

that Sections 4(1) and 9 of Article VIII simply mean that the President is required to fill vacancies in the courts within the time frames

provided therein unless prohibited by Section 15 of Article VII. It is not noteworthy that the prohibition on appointments comes into effect

only once every six years.

Section 15, Article VI is directed against two types of appointments: (1) those made for buying votes and (2) those made for

partisan considerations. The first refers to those appointments made within the two months preceding a Presidential election and are

similar to those which are declared elections offenses in the Omnibus Election Code, viz.:

Sec. 261. Prohibited Acts. — The following shall be guilty of an election offense:

(a) Vote-buying and vote-selling. — (1) Any person who gives, offer or promises money or anything of value  gives or promises any

office or employment, franchise or grant, public or private, or makes or offers to make an expenditure, directly or indirectly, or cause an

expenditure to be made to any person, association, corporation, entity, or community in order to induce anyone or the public in general

to vote for or against any candidate or withhold his vote in the election, or to vote for or against any aspirant for thenomination or choice

of a candidate in a convention or similar selection process of a political party.

xxx xxx xxx

(g) Appointment of new employees, creation of new position, promotion, or giving salary increases. — During the period of forty-five

days before a regular election and thirty days before a regular election and thirty days before a special election, (1) any head, official or

appointing officer of a government office, agency or instrumentality, whether national or local, including government-owned or

controlled corporations, who appoints or hiresany new employee, whether provisional, temporary, or casual, or creates and fills any

new position, except upon prior authority of the Commission. The Commission shall not grant the authority sought unless, it is satisfied

that the position to be filled is essential to the proper functioning of the office or agency concerned, and that the position shall not be

filled in a manner thatmay influence the election.

The second type of appointments prohibited by Section 15, Article VII consist of the so-called “midnight” appointments.

There may well be appointments to important positions which have to be made even after the proclamations of a new President. Such

Page 33: Article 7 Compiled Digested Cases

appointments, so long as they are “few and so spaced as to afford some assurance of deliberate action and careful consideration of the

need for the appointment and the appointee’s qualifications,” can be made by the outgoing President.

                Section 15 may not unreasonably be deemed to contemplate not only “midnight” appointments — those made obviously for

partisan reasons as shown by their number and the time of their making — but also appointments of the Presidential election.

The exception in the same Section 15 of Article VII allows only the making oftemporary appointments to executive positions

when continued vacancies will prejudice public service or endanger public safety. Obviously, the article greatly restricts the appointing

power of the President during the period of the ban.

Considering the respective reasons for the time frames for filling vacancies in the courts and the restriction on the President’s

power of appointments, it is the Supreme Court’s view that, as a general proposition, in case of conflict, the former should yield to the

latter. Surely, the prevention of vote-buying and similar evils outweighs the need for avoiding delays in filling up of court vacancies or

the disposition of some cases. Temporary vacancies can abide the period of the ban which, incidentally and as earlier pointed out,

comes to exist only once in every six years. Moreover, those occurring in the lower courts can be filled temporarily by designation. But

prohibited appointments are long-lasting and permanent in their effects. They may, as earlier pointed out, their making is considered an

election offense.

To be sure, instances may be conceived of the imperative need for an appointment, during the period of the ban, not only in

the executive but also in the Supreme Court. This may be the case should the membership of the Court be so reduced that it will have

no quorum, or should the voting on a particularly important question requiring expeditious resolution be evenly divided. Such a case,

however, is covered by neither Section 15 of Article VII nor Sections 4 (1) and 9 of Article VIII.

SECTION 16

GP V SPRINGER

50 Phil 259 – Law on Public Officers – Power to Appoint is Essentially Executive 

Sometime in the 1900s, the National Coal Company (NCC) was created by the Philippine Congress. The law created it (Act No. 2822)

provides that: “The voting power … shall be vested exclusively in a committee consisting of the Governor-General, the President of the

Senate, and the Speaker of the House of Representatives.”

In November 1926, the Governor-General (Leonard Wood) issued E.O. No. 37 which divested the voting rights of the Senate President

and House Speaker in the NCC. The EO emphasized that the voting right should be solely lodged in the Governor-General who is the

head of the government (President at that time was considered the head of state but does not manage government affairs). A copy of

the said EO was furnished to the Senate President and the House Speaker.

However, in December 1926, NCC held its elections and the Senate President as well as the House Speaker, notwithstanding EO No.

37 and the objection of the Governor-General,  still elected Milton Springer and four others as Board of Directors of NCC. Thereafter,

a quo warranto proceeding in behalf of the government was filed against Springer et al questioning the validity of their election into the

Board of NCC.

ISSUE: Whether or not the Senate President as well as the House Speaker can validly elect the Board Members of NCC.

HELD: No. E.O. No 37 is valid. It is in accordance with the doctrine of separation of powers. The Supreme Court emphasized that the

legislature creates the public office but it has nothing to do with designating the persons to fill the office. Appointing persons to a public

office is essentially executive. The NCC is a government owned and controlled corporation. It was created by Congress. To extend the

power of Congress into allowing it, through the Senate President and the House Speaker, to appoint members of the NCC is already an

invasion of executive powers. The Supreme Court however notes that indeed there are exceptions to this rule where the legislature

may appoint persons to fill public office. Such exception can be found in the appointment by the legislature of persons to fill offices

within the legislative branch – this exception is allowable because it does not weaken the executive branch.

BERMUDEZ V TORRES

FACTS:

The vacancy in the Office of the Provincial Prosecutor of Tarlac impelled the main contestants in this case, petitioner

Oscar Bermudez and respondent Conrado Quiaoit, to take contrasting views on the proper interpretation of a provision

in the 1987 Revised Administrative Code. Bermudez was a recommendee of then Justice Secretary Teofisto Guingona,

Jr., for the position of Provincial Prosecutor. Quiaoit, on the other hand, had the support of then Representative Jose

Yap. On 30 June 1997, President Ramos appointed Quiaoit to the coveted office. Quiaoit received a certified xerox copy

of his appointment and, on 21 July 1997, took his oath of office before Executive Judge Angel Parazo of the Regional

Trial Court (Branch 65) of Tarlac, Tarlac. On 23 July 1997, Quiaoit assumed office and immediately informed the

President, as well as the Secretary of Justice and the Civil Service Commission, of that assumption.

Page 34: Article 7 Compiled Digested Cases

On 10 October 1997, Bermudez filed with the Regional Trial Court of Tarlac, a petition for prohibition and/or injunction,

and mandamus, with a prayer for the issuance of a writ of injunction/temporary restraining order, against herein

respondents, challenging the appointment of Quiaoit primarily on the ground that the appointment lacks the

recommendation of the Secretary of Justice prescribed under the Revised Administrative Code of 1987. After hearing,

the trial court considered the petition submitted for resolution and, in due time, issued its now assailed order

dismissing the petition. The subsequent move by petitioners to have the order reconsidered met with a denial.

ISSUE:

Whether or not the absence of a recommendation of the Secretary of Justice to the President can be held fatal to the

appointment of respondent Conrado Quiaoit.

HELD:

The petition is denied. An “appointment” to a public office is the unequivocal act of designating or selecting by one

having the authority therefor of an individual to discharge and perform the duties and functions of an office or trust.

The appointment is deemed complete once the last act required of the appointing authority has been complied with

and its acceptance thereafter by the appointee in order to render it effective.

Indeed, it may rightly be said that the right of choice is the heart of the power to appoint. In the exercise of the power

of appointment, discretion is an integral part thereof.

When the Constitution or the law clothes the President with the power to appoint a subordinate officer, such

conferment must be understood as necessarily carrying with it an ample discretion of whom to appoint. It should be

here pertinent to state that the President is the head of government whose authority includes the power of control

over all “executive departments, bureaus and offices.”

It is the considered view of the Court that the phrase “upon recommendation of the Secretary,” found in Section 9,

Chapter II, Title III, Book IV, of the Revised Administrative Code, should be interpreted to be a mere advise, exhortation

or indorsement, which is essentially persuasive in character and not binding or obligatory upon the party to whom it is

made. The President, being the head of the Executive Department, could very well disregard or do away with the

action of the departments, bureaus or offices even in the exercise of discretionary authority, and in so opting, he

cannot be said as having acted beyond the scope of his authority.

FLORES V DRILON

FACTS:

The constitutionality of Sec. 13, par. (d), of R.A. 7227, otherwise known as the "Bases Conversion and Development Act of 1992," under which respondent Mayor Richard J. Gordon of Olongapo City was appointed Chairman and Chief Executive Officer of the Subic Bay Metropolitan Authority (SBMA), is challenged with prayer for prohibition, preliminary injunction and temporary restraining order. Said provision provides the President the power to appoint an administrator of the SBMA provided that in the first year of its operation, the Olongapo mayor shall be appointed as chairman and chief of executive of the Subic Authority. Petitioners maintain that such infringes to the constitutional provision of Sec. 7, first par., Art. IX-B, of the Constitution, which states that "no elective official shall be eligible for appointment or designation in any capacity to any public officer or position during his tenure," The petitioners also contend that Congress encroaches upon the discretionary power of the President to appoint.

ISSUE:

Whether or not said provision of the RA 7227 violates the constitutional prescription against appointment or designation of elective officials to other government posts.

RULING:

The court held the Constitution seeks to prevent a public officer to hold multiple functions since they are accorded with a public office that is a full time job to let them function without the distraction of other governmental duties.

Page 35: Article 7 Compiled Digested Cases

The Congress gives the President the appointing authority which it cannot limit by providing the condition that in the first year of the operation the Mayor of Olongapo City shall assume the Chairmanship. The court points out that the appointing authority the congress gives to the President is no power at all as it curtails the right of the President to exercise discretion of whom to appoint by limiting his choice.

BAUTISTA V CA

Political Law – Appointments – CHR

On 27 Aug 1987, Cory designated Bautista as the Acting Chairwoman of CHR. In December of the same year, Cory made the

designation of Bautista permanent.  The CoA, ignoring the decision in the Mison case, averred that Bautista cannot take her seat w/o

their confirmation. Cory, through the Exec Sec, filed with the CoA communications about Bautista’s appointment on 14 Jan 1989.

Bautista refused to be placed under the CoA’s review hence she filed a petition before the SC. On the other hand, Mallillin invoked EO

163-A stating that since CoA refused Bautista’s appointment, Bautista should be removed. EO 163-A provides that the tenure of the

Chairman and the Commissioners of the CHR should be at the pleasure of the President.

ISSUE: Whether or not Bautista’s appointment is subject to CoA’s confirmation.

HELD: Since the position of Chairman of the CHR is not among the positions mentioned in the first sentence of Sec. 16, Art. 7 of the

1987 Constitution, appointments to which are to be made with the confirmation of the CoA it follows that the appointment by the

President of the Chairman of the CHR is to be made without the review or participation of the CoA. To be more precise, the

appointment of the Chairman and Members of the CHR is not specifically provided for in the Constitution itself, unlike the Chairmen and

Members of the CSC, the CoE and the COA, whose appointments are expressly vested by the Constitution in the President with the

consent of the CoA. The President appoints the Chairman and Members of the CHR pursuant to the second sentence in Sec 16, Art. 7,

that is, without the confirmation of the CoA because they are among the officers of government “whom he (the President) may be

authorized by law to appoint.” And Sec 2(c), EO 163 authorizes the President to appoint the Chairman and Members of the CHR.

Because of the fact that the president submitted to the CoA on 14 Jan 1989 the appointment of Bautista, the CoA argued that the

president though she has the sole prerogative to make CHR appointments may from time to time ask confirmation with the CoA. This is

untenable according to the SC. The Constitution has blocked off certain appointments for the President to make with the participation of

the Commission on Appointments, so also has the Constitution mandated that the President can confer no power of participation in the

Commission on Appointments over other appointments exclusively reserved for her by the Constitution. The exercise of political options

that finds no support in the Constitution cannot be sustained. Further, EVEN IF THE PRESIDENT MAY VOLUNTARILY SUBMIT TO

THE COMMISSION ON APPOINTMENTS AN APPOINTMENT THAT UNDER THE CONSTITUTION SOLELY BELONGS TO HER,

STILL, THERE WAS NO VACANCY TO WHICH AN APPOINTMENT COULD BE MADE ON 14 JANUARY 1989. There can be no ad

interim appointments in the CHR for the appointment thereto is not subject to CoA’s confirmation. Appointments to the CHr is always

permanent in nature.

The provisions of EO 163-A is unconstitutional and cannot be invoked by Mallillin. The Chairman and the Commissioners of the CHR

cannot be removed at the pleasure of the president for it is constitutionally guaranteed that they must have a term of office.

CALDERON V CARALE

Political Law – Appointment – Cannot Be Expanded by Law

In 1989, RA 6715 was passed. This law amended PD 442 or the Labor Code. RA 6715 provides that the Chairman, the Division

Presiding Commissioners and other Commissioners [of the NLRC] shall all be appointed by the President, subject to confirmation by

the CoA. Appointments to any vacancy shall come from the nominees of the sector which nominated the predecessor. Pursuant to the

law, Cory assigned Carale et al as the Chairman and the Commissioners respectively of the NLRC, the appointment was not submitted

to the CoA for its confirmation. Calderon questioned the appointment saying that w/o the confirmation by the CoA, such an appointment

is in violation of RA 6715. Calderon asserted that RA 6715 is not an encroachment on the appointing power of the executive contained

in Sec16, Art. 7, of the Constitution, as Congress may, by law, require confirmation by the Commission on Appointments of other

officers appointed by the President additional to those mentioned in the first sentence of Sec 16 of Article 7 of the Constitution.

ISSUE: Whether or not Congress may, by law, require confirmation by the CoA of appointments extended by the President to

government officers additional to those expressly mentioned in the first sentence of Sec. 16, Art. 7 of the Constitution whose

appointments require confirmation by the CoA.

HELD: It is readily apparent that under the provisions of the 1987 Constitution, there are four (4) groups of officers whom the President

shall appoint. These four (4) groups are:

First, the heads of the executive departments, ambassadors, other public ministers and consuls, officers of the armed forces from the

rank of colonel or naval captain, and other officers whose appointments are vested in him in this Constitution;

Second, all other officers of the Government whose appointments are not otherwise provided for by law;

Third, those whom the President may be authorized by law to appoint;

Fourth, officers lower in rank whose appointments the Congress may by law vest in the President alone.

 

The SC agreed with the Sol-Gen, confirmation by the CoA is required exclusively for the heads of executive departments,

ambassadors, public ministers, consuls, officers of the armed forces from the rank of colonel or naval captain, and other officers whose

appointments are vested in the President by the Constitution, such as the members of the various Constitutional Commissions. With

respect to the other officers whose appointments are not otherwise provided for by the law and to those whom the President may be

authorized by law to appoint, no confirmation by the Commission on Appointments is required.

Page 36: Article 7 Compiled Digested Cases

“Had it been the intention to allow Congress to expand the list of officers whose appointments must be confirmed by the Commission on

Appointments, the Constitution would have said so by adding the phrase “and other officers required by law” at the end of the first

sentence, or the phrase, “with the consent of the Commission on Appointments” at the end of the second sentence. Evidently, our

Constitution has significantly omitted to provide for such additions.

 

Jurisprudence established the following in interpreting Sec 16, Art 7 of the Constitution

1.  Confirmation by the Commission on Appointments is required only for presidential appointees mentioned in the first sentence of

Section 16, Article VII, including, those officers whose appointments are expressly vested by the Constitution itself in the president (like

sectoral representatives to Congress and members of the constitutional commissions of Audit, Civil Service and Election).

2.  Confirmation is not required when the President appoints other government officers whose appointments are not otherwise provided

for by law or those officers whom he may be authorized by law to appoint (like the Chairman and Members of the Commission on

Human Rights). Also, as observed in Mison, when Congress creates inferior offices but omits to provide for appointment thereto, or

provides in an unconstitutional manner for such appointments, the officers are considered as among those whose appointments are not

otherwise provided for by law.

DELEZ V CONSTI COMMISSION

FACTS:

The petitioner and three others were appointed Sectoral Representatives by the President pursuant to Article VII,

Section 16, paragraph 2 and Article XVIII, Section 7 of the Constitution. Due to the opposition of some congressmen-

members of the Commission on Appointments, who insisted that sectoral representatives must first be confirmed by

the respondent Commission before they could take their oaths and/or assume office as members of the House of

Representatives, Speaker Ramon V. Mitra, Jr. suspended the oathtaking of the four sectoral representatives which was

scheduled at the Session Hall of Congress after the Order of Business.

In view of this development, Executive Secretary Catalino Macaraig, Jr. transmitted on April 25, 1988, a letter dated

April 11, 1988 of the President addressed to the Commission on Appointments submitting for confirmation the

appointments of the four sectoral representatives.

Meanwhile, petitioner in a letter dated April 22, 1988 addressed to Speaker Ramon V. Mitra, Jr. (Annex V) appealed to

the House of Representatives alleging, among others, that since “no attempt was made to subject the sectoral

representatives ** already sitting to the confirmation process, there is no necessity for such confirmation and

subjection thereto of the present batch would certainly be discriminatory.”

On May 10, 1988, petitioner Deles received an invitation from the Commission on Appointments for the deliberation of

her appointment as sectoral representative for women. Petitioner sent a reply dated May 11, 1988 explaining her

position and questioning the jurisdiction of the Commission on Appointments over the appointment of sectoral

representatives.

In the May 12, 1988 meeting of the Committee of the Constitutional Commissions and Offices of the Commission on

Appointments, chaired by Sen. Edgardo J. Angara, the Committee ruled against the position of petitioner Deles.

ISSUE

Does the Constitution require the appointment of sectoral representatives to the House of Representatives to be

confirmed by the Commission on Appointments?

HELD:

The petition is dismissed. The power to appoint is fundamentally executive or presidential in character. Since the seats

reserved for sectoral representatives in paragraph 2, Section 5, Art. VI may be filled by appointment by the President

by express provision of Section 7, Art. XVIII of the Constitution, it is undubitable that sectoral representatives to the

House of Representatives are among the “other officers whose appointments are vested in the President in this

Constitution,” referred to in the first sentence of Section 16, Art. VII whose appointments are-subject to confirmation

by the Commission on Appointments (Sarmiento v. Mison, supra).

Petitioner’s appointment was furthermore made pursuant to Art. VII, Section 16, paragraph 2 which gives the President

”the power to make appointments during the recess of the Congress, whether voluntary or compulsory, but such

appointments shall be effective only until disapproval by the Commission on Appointments or until the next

adjournment of the Congress.” The records show that petitioner’s appointment was made on April 6, 1988 or while

Page 37: Article 7 Compiled Digested Cases

Congress was in recess (March 26, 1988 to April 17, 1988); hence, the reference to the said paragraph 2 of Section 16,

Art. VII in the appointment extended to her.

Implicit in the invocation of paragraph 2, Section 16, Art. VII as authority for the appointment of petitioner is, the

recognition by the President as appointing authority, that petitioner’s appointment requires confirmation by the

Commission on Appointments. As a matter of fact, the President had expressly submitted petitioner’s appointment for

confirmation by the Commission on Appointments. Considering that Congress had adjourned without respondent

Commission on Appointments having acted on petitioner’s appointment, said appointment/nomination had become

moot and academic pursuant to Section 23 of the Rules of respondent Commission and “unless resubmitted shall not

again be considered by the Commission.”

MANALO V SISTOZA

Appointments

Petitioner, Jesulito Sistoza question the constitutionality and legality of the appointments issued by former Pres. Corazon Aquino to the

respondent senior officers of the PNP who were promoted to the rank of Chief Superintendent and Director without their appointments

submitted to the Commission on Appointments for confirmation. The said police officers tool their Oath of Offices and assumed their

respective positions. Thereafter, the Department of Budget and Management, under the then Secretary Salvador Enriquez III,

authorized disbursements for their salaries and other emoluments. The petitioner brought before this petition for prohibition, as a tax

payer suit to the SC to assail the legality of subject appointment and disbursement thereof.

ISSUE: Whether or not the appointment of the senior officers of the PNP is valid even without the confirmation of the Commission on

Appointments.

HELD: The SC held that the appointments are valid. The court has the inherent authority to determine whether a statute enacted by the

legislature transcends the limit alienated by the fundamental law. When it does the courts will not hesitate to strike down such

unconstitutionality.

Matibag vs Benipayo

GR No. 149036

April 2, 2002

Maria J. Angelina G. Matibag questions the constitutionality of the appointment by President Arroyo of Benipayo (Chairman of the

Commission on Elections), and Bora and Tuason (COMELEC Commissioners). She questions the legality of appointment by Benipayo

of Velma J. Cinco as Director IV of the Comelec’s EID and reassigning her to the Law department.

Issues:

1.    Instant petition satisfies all requirements

2.    Assumption of office by Benipayo, Bora and Tuason; ad interim appointments amounts to a temporary appointment prohibited by Sec 1

(2), Article IX-C of the Constitution

3.    Renewal of ad interim violated the prohibition on reappointment under Sec 1 (2), Article IX-C of the Constitution

4.    Benipayo’s removal of petitioner is illegal

5.    OIC of COMELEC’s Finance Services Department acting in excess jurisdiction

Matibag’s Argument:

1.    Failure to consult for reassignment

2.    Civil Service Commission Memorandum Circular No 7; transferring and detailing employees are prohibited during the election period

beginning January 2 until June 13, 2001

3.    Reassignment violated Sec 261 of the Omnibus Election Code, COMELEC Resolution No. 3258

4.    Ad interim appointments of Benipayo, Bora and Tuason violated the constitutional provisions on the independence of the COMELEC

5.    Illegal removal or reassignment

6.    Challenges the designation of Cinco

7.    Questions the disbursement made by COMELEC

8.    No ad interim appointment to the COMELEC or to Civil Service Commission and COA

9.    Sec 1 (2) of Article IX-C; an ad interim appointee cannot assume office until confirmed by the Commission on Appointments

Benipayo’s Argument:

1.    Comelec Resolution No. 3300

2.    Petitioner does not have personal interest, not directly injured

3.    Failure to question constitutionality of ad interim appointments at the earliest opportunity. She filed only after third time of

reappointments

4.    Ad interim is not the lis mota because the real issue is the legality of petitioner’s reassignment.

Rules of Court:

1.    Real issue is whether or not Benipayo is the lawful Chairman of the Comelec

2.    Petitioner has a personal and material stake.

3.    It is not the date of filing of the petition that determines whether the constitutional issue was raised at the earliest point. The earliest

opportunity to raise a constitutional issue is to raise it in the pleading.

Page 38: Article 7 Compiled Digested Cases

4.    Questioned the constitutionality of the ad interim appointments which is the earliest opportunity for pleading the constitutional issue

before a competent body.

5.    Ad interim appointment is a permanent appointment because it takes effect immediately and can no longer be withdrawn. It is not the

nature of appointment but the manner on which appointment was made. It will avoid interruptions that would result to prolonged

vacancies. It is limited the evil sought to be avoided.

6.    Termination of Ad interim appointment (Sword of Damocles); (1) disapproval (2) recess

7.    Two modes of appointment: (1) in session (2) in recess

8.    By-passed appointments – (1) lack of time/failure of the Commission on Appointments to organize, (2) subject of reconsideration, (3)

can be revived since there is no final disapproval

9.    Four situations in for a term of seven years without replacement: (1) serves his full seven-year term, (2) serves a part of his term and

then resigns before his seven-year term, (3) served the unexpired term of someone who died or resigned, (4) served a term of less than

seven years, and a vacancy arises from death or resignation. Not one of the four situation applies to the case of Benipayo, Borra or

Tuason

10. Reappointment cannot be applied; (1) appointed by president, (2) confirmed by Commission on Appointments

11. Without reappointment means: (first phrase) prohibits reappointment of any person previously appointed for a term of seven

years (second phrase) prohibits reappointment of any person previously appointed for a term of 5 or 3 years pursuant to the first set of

appointees

12. Reasons for prohibition of reappointments: (1) prevent second appointment (2) not serve beyond the fixed term

13. Two important amendments: (1) requiring the consent by Commission of Appointments (2) prohibition on serving beyond the fixed term

of 7 years

14. Twin Prohibition (ironclad): (1) prohibition of reappointments (2) prohibition of temporary or acting appointments

15. Third issue not violation because the previous appointments were not confirmed by the Commission on Appointments.

16. Benipayo is the de jure COMELEC Chairman. He is not required by law to secure the approval of the COMELEC en banc.

17. The petitioner is acting only temporary because a permanent appointment can be issued only upon meeting all the requirements.COMELEC Resolution No. 3300 refers only to COMELEC field personnel not to head office personnel.

Rufino vs. Endriga, G.R. No. 139554, July 21, 2006

Appointment Power of President

Under Section 16, Article VII of the 1987 Constitution, the President appoints three groups of officers. The first group refers to the heads of the Executive departments, ambassadors, other public ministers and consuls, officers of the armed forces from the rank of colonel or naval captain, and other officers whose appointments are vested in the President by the Constitution. The second group refers to those whom the President may be authorized by law to appoint. The third group refers to all other officers of the Government whose appointments are not otherwise provided by law.

Under the same Section 16, there is a fourth group of lower-ranked officers whose appointments Congress may by law vest in the heads of departments, agencies, commissions, or boards. The present case involves the interpretation of Section 16, Article VII of the 1987 Constitution with respect to the appointment of this fourth group of officers.

The President appoints the first group of officers with the consent of the Commission on Appointments. The President appoints the second and third groups of officers without the consent of the Commission on Appointments. The President appoints the third group of officers if the law is silent on who is the appointing power, or if the law authorizing the head of a department, agency, commission, or board to appoint is declared unconstitutional. Thus, if Section 6(b) and (c) of PD 15 is found unconstitutional, the President shall appoint the trustees of the CCP Board because the trustees fall under the third group of officers.

Scope of Appointment Power of the Heads of Departments, Agencies, Commissions or Boards

The framers of the 1987 Constitution clearly intended that Congress could by law vest the appointment of lower-ranked officers in the heads of departments, agencies, commissions, or boards. The deliberations of the 1986 Constitutional Commission explain this intent beyond any doubt.

The framers of the 1987 Constitution changed the qualifying word “inferior” to the less disparaging phrase “lower in rank” purely for style. However, the clear intent remained that these inferior or lower in rank officers are the subordinates of the heads of departments,

Page 39: Article 7 Compiled Digested Cases

agencies, commissions, or boards who are vested by law with the power to appoint. The express language of the Constitution and the clear intent of its framers point to only one conclusion — the officers whom the heads of departments, agencies, commissions, or boards may appoint must be of lower rank than those vested by law with the power to appoint.

Congress may vest the authority to appoint only in the heads of the named offices

Further, Section 16, Article VII of the 1987 Constitution authorizes Congress to vest “in the heads of departments, agencies, commissions, or boards” the power to appoint lower-ranked officers. xxx

In a department in the Executive branch, the head is the Secretary. The law may not authorize the Undersecretary, acting as such Undersecretary, to appoint lower-ranked officers in the Executive department. In an agency, the power is vested in the head of the agency for it would be preposterous to vest it in the agency itself. In a commission, the head is the chairperson of the commission. In a board, the head is also the chairperson of the board. In the last three situations, the law may not also authorize officers other than the heads of the agency, commission, or board to appoint lower-ranked officers.

The grant of the power to appoint to the heads of agencies, commissions, or boards is a matter of legislative grace. Congress has the discretion to grant to, or withhold from, the heads of agencies, commissions, or boards the power to appoint lower-ranked officers. If it so grants, Congress may impose certain conditions for the exercise of such legislative delegation, like requiring the recommendation of subordinate officers or the concurrence of the other members of the commission or board.

This is in contrast to the President’s power to appoint which is a self-executing power vested by the Constitution itself and thus not subject to legislative limitations or conditions. The power to appoint conferred directly by the Constitution on the Supreme Court en banc and on the Constitutional Commissions is also self-executing and not subject to legislative limitations or conditions.

The Constitution authorizes Congress to vest the power to appoint lower-ranked officers specifically in the “heads” of the specified offices, and in no other person. The word “heads” refers to the chairpersons of the commissions or boards and not to their members xxx.

President’s Power of Control

The presidential power of control over the Executive branch of government extends to all executive employees from the Department Secretary to the lowliest clerk. This constitutional power of the President is self-executing and does not require any implementing law. Congress cannot limit or curtail the President’s power of control over the Executive branch.

xxx

The CCP does not fall under the Legislative or Judicial branches of government. The CCP is also not one of the independent constitutional bodies. Neither is the CCP a quasi-judicial body nor a local government unit. Thus, the CCP must fall under the Executive branch. Under the Revised Administrative Code of 1987, any agency “not placed by law or order creating them under any specific department” falls “under the Office of the President.”

Since the President exercises control over “all the executive departments, bureaus, and offices,” the President necessarily exercises control over the CCP which is an office in the Executive branch. In mandating that the President “shall have control of all executive x x x offices,” Section 17, Article VII of the 1987 Constitution does not exempt any executive office — one performing executive functions outside of the independent constitutional bodies — from the President’s power of control. There is no dispute that the CCP performs executive, and not legislative, judicial, or quasi-judicial functions.

The President’s power of control applies to the acts or decisions of all officers in the Executive branch. This is true whether such officers are appointed by the President or by heads of departments, agencies, commissions, or boards. The power of control means the power to revise or reverse the acts or decisions of a subordinate officer involving the exercise of discretion.

In short, the President sits at the apex of the Executive branch, and exercises “control of all the executive departments, bureaus, and offices.” There can be no instance under the Constitution where an officer of the Executive branch is outside the control of the President. The Executive branch is unitary since there is only one President vested with executive power exercising control over the

Page 40: Article 7 Compiled Digested Cases

entire Executive branch. Any office in the Executive branch that is not under the control of the President is a lost command whose existence is without any legal or constitutional basis.

The Legislature cannot validly enact a law that puts a government office in the Executive branch outside the control of the President in the guise of insulating that office from politics or making it independent. If the office is part of the Executive branch, it must remain subject to the control of the President. Otherwise, the Legislature can deprive the President of his constitutional power of control over “all the executive x x x offices.” If the Legislature can do this with the Executive branch, then the Legislature can also deal a similar blow to the Judicial branch by enacting a law putting decisions of certain lower courts beyond the review power of the Supreme Court. This will destroy the system of checks and balances finely structured in the 1987 Constitution among the Executive, Legislative, and Judicial branches.

Of course, the President’s power of control does not extend to quasi-judicial bodies whose proceedings and decisions are judicial in nature and subject to judicial review, even as such quasi-judicial bodies may be under the administrative supervision of the President. It also does not extend to local government units, which are merely under the general supervision of the President

PIMENTEL V ERMITA

472 SCRA 587 – Political Law – Commission on Appointment – Ad Interim Appointments vs Appointments in an Acting Capacity

Law on Public Officers – Modes and Kinds of Appointment

While Congress was in session, due to vacancies in the cabinet, then president Gloria Macapagal-Arroyo (GMA) appointed Arthur Yap

et al as secretaries of their respective departments. They were appointed in an acting capacity only. Senator Aquilino Pimentel together

with 7 other senators filed a complaint against the appointment of Yap et al. Pimentel averred that GMA cannot make such appointment

without the consent of the Commission on Appointment; that, in accordance with Section 10, Chapter 2, Book IV of Executive Order No.

292, only the undersecretary of the respective departments should be designated in an acting capacity and not anyone else.

On the contrary, then Executive Secretary Eduardo Ermita averred that the president is empowered by Section 16, Article VII of the

1987 Constitution to issue appointments in an acting capacity to department secretaries without the consent of the Commission on

Appointments even while Congress is in session. Further, EO 292 itself allows the president to issue temporary designation to an officer

in the civil service provided that the temporary designation shall not exceed one year.

During the pendency of said case, Congress adjourned and GMA issued ad interimappointments re-appointing those previously

appointed in acting capacity.

ISSUE: Whether or not the appointments made by ex PGMA is valid.

HELD: Yes. The argument raised by Ermita is correct. Further, EO 292 itself provided the safeguard so that such power will not be

abused hence the provision that the temporary designation shall not exceed one year. In this case, in less than a year after the initial

appointments made by GMA, and when the Congress was in recess, GMA issued the ad interim appointments – this also proves that

the president was in good faith.

It must also be noted that cabinet secretaries are the alter egos of the president. The choice is the president’s to make and the

president normally appoints those whom he/she can trust. She cannot be constrained to choose the undersecretary. She has the option

to choose. An alter ego, whether temporary or permanent, holds a position of great trust and confidence. Congress, in the guise of

prescribing qualifications to an office, cannot impose on the President who her alter ego should be.

The office of a department secretary may become vacant while Congress is in session. Since a department secretary is the alter ego of

the President, the acting appointee to the office must necessarily have the President’s confidence. That person may or may not be the

permanent appointee, but practical reasons may make it expedient that the acting appointee will also be the permanent appointee.

Anent the issue that GMA appointed “outsiders”, such is allowed. EO 292 also provides that the president “may temporarily designate

an officer already in the government service or any other competent person to perform the functions of an office in the executive

branch.” Thus, the President may even appoint in an acting capacity a person not yet in the government service, as long as the

President deems that person competent.

SECTION 17

LACSON V EXECUTIVE SECRETARY

Political Law – Delegation of Control Power to the Executive Secretary

Magallanes was permitted to use and occupy a land used for pasture in Davao. The said land was a forest zone which was later

declared as an agricultural zone. Magallanes then ceded his rights to LMC of which he is a co-owner.  Paño was a farmer who asserted

his claim over the same piece of land. The Director of Lands denied Paño’s request.  The Secretary of Agriculture likewise denied his

petition hence it was elevated to the Office of the President. Exec Sec Pajo ruled in favor of Paño. LMC averred that the earlier decision

of the Secretary is already conclusive hence beyond appeal. He also averred that  the decision of the Executive Secretary is an undue

delegation of power. The Constitution, LMC asserts, does not contain any provision whereby the presidential power of control may be

delegated to the Executive Secretary. It is argued that it is the constitutional duty of the President to act personally upon the matter.

ISSUE: Whether or not the power of control may be delegated to the Exec Sec and may it be further delegated by the Exec Sec.

Page 41: Article 7 Compiled Digested Cases

HELD: The President’s duty to execute the law is of constitutional origin.    So, too, is his control of all executive departments.    Thus it

is, that department heads are men of his confidence. His is the power to appoint them; his, too, is the privilege to dismiss them at

pleasure. Naturally, he controls and directs their acts. Implicit then is his authority to go over, confirm, modify or reverse the action

taken by his department secretaries. In this context, it may not be said that the President cannot rule on the correctness of a decision of

a department secretary. Parenthetically, it may be stated that the right to appeal to the President reposes upon the President’s power of

control over the executive departments.   And control simply means “the power of an officer to alter or modify or nullify or set aside what

a subordinate officer had done in the performance of his duties and to substitute the judgment of the former for that of the latter.”

It is correct to say that constitutional powers there are which the President must exercise in person.    Not as correct, however, is it to

say that the Chief Executive may not delegate to his Executive Secretary acts which the Constitution does not command that he

perform in person.    Reason is not wanting for this view. The President is not expected to perform in person all the multifarious

executive and administrative functions. The office of the Executive Secretary is an auxiliary unit which assists the President. The rule

which has thus gained recognition is that “under our constitutional setup the Executive Secretary who acts for and in behalf and by

authority of the President has an undisputed jurisdiction to affirm, modify, or even reverse any order” that the Secretary of Agriculture

and Natural Resources, including the Director of Lands, may issue.

Maceda v Macaraig

Facts:

The petition seeks to nullify certain decisions, orders, ruling, and resolutions of the respondents (Macaraig et. al) for exempting the National Power Corporation (NPC) from indirect tax and duties. Commonwealth Act 120 created NPC as a public corporation. RA 6395 revised the charter of NPC and provided in detail the exemption of NPC from all taxes, duties and other charges by the government. There were many resolutions and decisions that followed after RA 6395 which talked about the exemption and non-exemption from taxes of NPC.

Issue:

Whether or not NPC is really exempt from indirect taxes

Held:

Yes. NPC is a non-profit public corporation created for the general good and welfare of the people. From the very beginning of its corporate existence, NPC enjoyed preferential tax treatment to enable it to pay its debts and obligations. From the changes made in the NPC charter, the intention to strengthen its preferential tax treatment is obvious. The tax exemption is intended not only to insure that the NPC shall continue to generate electricity for the country but more importantly, to assure cheaper rates to be paid by consumers.

------------------

Some Notes on Direct and Indirect Taxes:

Direct Taxes – those which a taxpayer is directly liable on the transaction or business it engages in. Examples are: custom duties, ad valorem taxes paid by oil companies for importation of crude oil

Indirect Taxes – paid by persons who can shift the burden upon someone else.

Examples are: ad valorem taxes that oil companies pay to BIR upon removal of petroleum products from its refinery can be shifted to its buyer, like the NPC

Dissenting Opinion of Justice Sarmiento: The fact that NPC has been tasked with the enormous undertaking to improve the quality of life, is no reason, to include indirect taxes, within the coverage of its preferential tax treatment. The deletion of “indirect taxes” as stated in one of the assailed orders (PD 938), is significant, because if said law truly intends to exempt NPC from indirect taxes, it would have said so specifically.

Article VII, Section 21

VINUYA VS. SEC. ROMULO

ISABELITA C. VINUYA, VICTORIA C. DELA PEÑA, HERMINIHILDA, MANIMBO, LEONOR H. SUMAWANG, CANDELARIA L. SOLIMAN, MARIA L. QUILANTANG, MARIA L. MAGISA, NATALIA M. ALONZO, LOURDES M. NAVARO, FRANCISCA M. ATENCIO, ERLINDA MANALASTAS, TARCILA M. SAMPANG, ESTER M. PALACIO MAXIMA R. DELA CRUZ, BELEN A. SAGUM, FELICIDAD TURLA, FLORENCIA M. DELA PEÑA, FRANCIA A. BUCO, PASTORA C. GUEVARRA, VICTORIA M. DELA CRUZ, PETRONILA O. DELA CRUZ, ZENAIDA P. DELA CRUZ, CORAZON M. SUBA, EMERINCIANA A. VINUYA, LYDIA A. SANCHEZ, ROSALINA M.BUCO, PATRICIA A. ERNARDO, LUCILA H. PAYAWAL, MAGDALENA LIWAG, ESTER C. BALINGIT, JOVITA A. DAVID, EMILIA C. MANGILIT, VERGINIA M. BANGIT, GUILLERMA S. BALINGIT, TERECITA PANGILINAN, MAMERTA C. PUNO, CRISENCIANA C.

Page 42: Article 7 Compiled Digested Cases

GULAPA, SEFERINA S. TURLA, MAXIMA B. TURLA, LEONICIA G. GUEVARRA, ROSALINA M. CULALA, CATALINA Y. MANIO, MAMERTA T. SAGUM, CARIDAD L. TURLA, et al.

In their capacity and as members of the “Malaya Lolas Organization”,

– versus -

THE HONORABLE EXECUTIVE SECRETARY ALBERTO G. ROMULO, THE HONORABLE SECRETARY OF FOREIGNAFFAIRS DELIA DOMINGO-ALBERT, THE HONORABLE SECRETARY OF JUSTICE MERCEDITAS N. GUTIERREZ, and THE HONORABLE SOLICITOR GENERAL ALFREDO L. BENIPAYO

G.R. No. 162230, April 28, 2010

FACTS:

This is an original Petition for Certiorari under Rule 65 of the Rules of Court with an application for the issuance of a writ of preliminary mandatory injunction against the Office of the Executive Secretary, the Secretary of the DFA, the Secretary of the DOJ, and the OSG.

Petitioners are all members of the MALAYA LOLAS, a non-stock, non-profit organization registered with the SEC, established for the purpose of providing aid to the victims of rape by Japanese military forces in the Philippines during the Second World War.

Petitioners claim that since 1998, they have approached the Executive Department through the DOJ, DFA, and OSG, requesting assistance in filing a claim against the Japanese officials and military officers who ordered the establishment of the “comfort women” stations in the Philippines. But officials of the Executive Department declined to assist the petitioners, and took the position that the individual claims of the comfort women for compensation had already been fully satisfied by Japan’s compliance with the Peace Treaty between the Philippines and Japan.

Hence, this petition where petitioners pray for this court to (a) declare that respondents committed grave abuse of discretion amounting to lack or excess of discretion in refusing to espouse their claims for the crimes against humanity and war crimes committed against them; and (b) compel the respondents to espouse their claims for official apology and other forms of reparations against Japan before the International Court of Justice (ICJ) and other international tribunals.

Respondents maintain that all claims of the Philippines and its nationals relative to the war were dealt with in the San Francisco Peace Treaty of 1951 and the bilateral Reparations Agreement of 1956.

On January 15, 1997, the Asian Women’s Fund and the Philippine government signed a Memorandum of Understanding for medical and welfare support programs for former comfort women. Over the next five years, these were implemented by the Department of Social Welfare and Development.

ISSUE:

WON the Executive Department committed grave abuse of discretion in not espousing petitioners’ claims for official apology and other forms of reparations against Japan.

RULING:

Petition lacks merit. From a Domestic Law Perspective, the Executive Department has the exclusive prerogative to determine whether to espouse petitioners’ claims against Japan.

Page 43: Article 7 Compiled Digested Cases

Political questions refer “to those questions which, under the Constitution, are to be decided by the people in their sovereign capacity, or in regard to which full discretionary authority has been delegated to the legislative or executive branch of the government. It is concerned with issues dependent upon the wisdom, not legality of a particular measure.”

One type of case of political questions involves questions of foreign relations. It is well-established that “the conduct of the foreign relations of our government is committed by the Constitution to the executive and legislative–’the political’–departments of the government, and the propriety of what may be done in the exercise of this political power is not subject to judicial inquiry or decision.” are delicate, complex, and involve large elements of prophecy. They are and should be undertaken only by those directly responsible to the people whose welfare they advance or imperil.

But not all cases implicating foreign relations present political questions, and courts certainly possess the authority to construe or invalidate treaties and executive agreements. However, the question whether the Philippine government should espouse claims of its nationals against a foreign government is a foreign relations matter, the authority for which is demonstrably committed by our Constitution not to the courts but to the political branches. In this case, the Executive Department has already decided that it is to the best interest of the country to waive all claims of its nationals for reparations against Japan in the Treaty of Peace of 1951. The wisdom of such decision is not for the courts to question.

The President, not Congress, has the better opportunity of knowing the conditions which prevail in foreign countries, and especially is this true in time of war. He has his confidential sources of information. He has his agents in the form of diplomatic, consular and other officials.

The Executive Department has determined that taking up petitioners’ cause would be inimical to our country’s foreign policy interests, and could disrupt our relations with Japan, thereby creating serious implications for stability in this region. For the to overturn the Executive Department’s determination would mean an assessment of the foreign policy judgments by a coordinate political branch to which authority to make that judgment has been constitutionally committed.

From a municipal law perspective, certiorari will not lie. As a general principle, where such an extraordinary length of time has lapsed between the treaty’s conclusion and our consideration – the Executive must be given ample discretion to assess the foreign policy considerations of espousing a claim against Japan, from the standpoint of both the interests of the petitioners and those of the Republic, and decide on that basis if apologies are sufficient, and whether further steps are appropriate or necessary.

In the international sphere, traditionally, the only means available for individuals to bring a claim within the international legal system has been when the individual is able to persuade a government to bring a claim on the individual’s behalf. By taking up the case of one of its subjects and by resorting to diplomatic action or international judicial proceedings on his behalf, a State is in reality asserting its own right to ensure, in the person of its subjects, respect for the rules of international law.

Within the limits prescribed by international law, a State may exercise diplomatic protection by whatever means and to whatever extent it thinks fit, for it is its own right that the State is asserting. Should the natural or legal person on whose behalf it is acting consider that their rights are not adequately protected, they have no remedy in international law. All they can do is resort to national law, if means are available, with a view to furthering their cause or obtaining redress. All these questions remain within the province of municipal law and do not affect the position internationally.

Even the invocation of jus cogens norms and erga omnes obligations will not alter this analysis. Petitioners have not shown that the crimes committed by the Japanese army violated jus cogens prohibitions at the time the Treaty of Peace was signed, or that the duty to prosecute perpetrators of international crimes is an erga omnes obligation or has attained the status of jus cogens.

The term erga omnes (Latin: in relation to everyone) in international law has been used as a legal term describing obligations owed by States towards the community of states as a whole. Essential distinction should be drawn between the obligations of a State towards the international community as a whole, and those arising vis-à-vis another State in the field of diplomatic protection. By their very nature, the former are the concern of all States. In view of the importance of the rights involved, all States can be held to have a legal interest in their protection; they are obligations erga omnes.

The term “jus cogens” (literally, “compelling law”) refers to norms that command peremptory authority, superseding conflicting treaties and custom. Jus cogens norms are considered peremptory in the sense that they are mandatory, do not admit derogation, and can be modified only by general international norms of equivalent authority

WHEREFORE, the Petition is hereby DISMISSED.

Page 44: Article 7 Compiled Digested Cases